Download as pdf or txt
Download as pdf or txt
You are on page 1of 83

Copyright 2010 - 2022 Maria Miller

EDITION 2/2022

All rights reserved. No part of this book may be reproduced or transmitted in any form or by any means, electronic or
mechanical, or by any information storage and retrieval system, without permission in writing from the author.

Copying permission: For having purchased this book, the copyright owner grants to the teacher-purchaser a limited
permission to reproduce this material for use with his or her students. In other words, the teacher-purchaser MAY make
copies of the pages, or an electronic copy of the PDF file, and provide them at no cost to the students he or she is actually
teaching, but not to students of other teachers. This permission also extends to the spouse of the purchaser, for the purpose
of providing copies for the children in the same family. Sharing the file with anyone else, whether via the Internet or other
media, is strictly prohibited.

No permission is granted for resale of the material.

The copyright holder also grants permission to the purchaser to make electronic copies of the material for back-up purposes.

If you have other needs, such as licensing for a school or tutoring center, please contact the author at
https://www.MathMammoth.com/contact.php

2 Percent (Blue Series)


Math Mammoth Percent
Contents

Introduction .................................................................... 5

Percent ............................................................................ 7
What Percentage…? ..................................................... 11
Percentage of a Number (Mental Math) ..................... 13
Percentage of a Number: Using Decimals ................... 16
Discounts ........................................................................ 19
Practice with Percent .................................................... 21
Finding the Total When the Percentage Is Known ... 24
Review: Percent ............................................................ 26
Percentage Basics ......................................................... 28
Solving Basic Percentage Problems ............................ 31
Percent Equations ........................................................ 34
Circle Graphs ............................................................... 39
Percentage of Change ................................................... 41
Percentage of Change: Applications ........................... 44
Comparing Values Using Percentages ........................ 48
Simple Interest .............................................................. 52
Final Review ......................................................... 58

Answers ......................................................................... 61

More from Math Mammoth ......................................... 83

3 Percent (Blue Series)


4 Percent (Blue Series)
Introduction
Math Mammoth Percent teaches students how to understand the concept of percent, to calculate the percentage
of a number, to figure discounts and sales tax, to draw circle graphs, to differentiate between a percentage of
change and a percentage of comparison, and to know how to calculate both. The text is suitable for grades 6
through 8, in what is often termed “middle school.”
The concept of percent builds on the student’s understanding of fractions and decimals. Specifically, students
should be very familiar with the idea of finding a fractional part of a whole (such as finding 3/4 of
$240. Students who have used Math Mammoth have been practicing that concept since 4th grade. One reason
why I have emphasized finding a fractional part of a whole so much in the earlier grades is specifically to lay a
groundwork for the concept of percent. Assuming the student has mastered how to find a fractional part of a
whole, and can easily convert fractions to decimals, then studying the concept of percent should not be difficult.
The first lesson, Percent, practices the concept of percent as a hundredth part, and how to write fractions and
decimals as percentages. Next, we study how to find a percentage, when the part and the whole are given (for
example, if 15 out of 25 club members are girls, what percentage of them are girls?).
The following two lessons have to do with finding a certain percentage of a given number or quantity. First, we
study how to do that using mental math techniques. For example, students find 10% of $400 by dividing $400 by
10. Next, students find a percentage of a quantity using decimal multiplication, including using a calculator. For
example, students find 17% of 45 km by multiplying 0.17 × 45 km.
I prefer teaching students to calculate percentages of quantities using decimals, instead of using percentage
proportion or some other method (such as changing 17% into the fraction 17/100 for calculations). That is
because using decimals is simpler: we simply change the percentage into a decimal, and multiply, instead of
having to build a proportion or use fractions. Also, decimals will be so much easier to use later on for solving
word problems that require the usage of equations.
Next is a lesson about discounts, which is an important application in everyday life. Then, we go on to the lesson
Practice with Percent, which contrasts the two types of problems students have already studied: questions that
ask for a certain percentage of a number (the percentage is given), and questions that ask for the percentage. For
example, the first type of question could be “What is 70% of $380?”, and the second type could be “What
percentage is $70 of $380?”
Finding the Total When the Percentage Is Known lets students find the total when the percentage and the partial
amount are known. For example: “Three-hundred twenty students, which is 40% of all students, take PE. How
many students are there in total?” We solve these with the help of bar models.
After a review lesson in the middle of the book, we study some of the basics again in the lessons Percentage and
Solving Basic Percentage Problems. While the concepts are the same as in the lessons in the beginning of this
book, this time we include more decimal digits and the coverage is faster, as these two lessons were originally
written for 7th grade.
Percent Equations is meant for pre-algebra students and covers how to solve basic percent problems using an
equation. It also explains the usage of a percent proportion.
The next major topic is the percentage of change, which is covered in a two-lesson sequence. The concept of
percentage of change deals with percentage increases and decreases in quantities (especially prices). For
example: “If an airline ticket that costs $120 now goes up by 10%, then what will the new price be?” Students
will also learn how to find an unknown percentage of change when the original and new quantities are known.
For example, “If a shirt cost $24 and is now discounted to $18, then what percentage was the discount?”
Tying in with percentage of change, there is one lesson on Comparing Values Using Percentages. Students learn
to solve comparisons involving percent (such as how many percent more (or less) one thing is than another)
through applying concepts that they learned in finding the percentage of change and to differentiate clearly
among the various types of comparison questions that can be asked.

5 Percent (Blue Series)


Simple Interest is a lesson on the important topic of interest, using as a context both loans and savings accounts.
Students learn to use the formula I = prt in a great variety of problems and situations.
The text concludes with a thorough review lesson of all of the concepts taught in the other lessons.
I have made several videos to match the lessons from this book. You can watch them here:
http://www.mathmammoth.com/videos/percent

Wishing you success in your endeavors to teach math,


Maria Miller, the author

Helpful Resources on the Internet


We have compiled a list of external Internet resources that match the topics in this book. This list of links
includes web pages that offer:

• online practice for concepts;


• online games, or occasionally, printable games;
• animations and interactive illustrations of math concepts;
• articles that teach a math concept.

We heartily recommend you take a look at the list. Many of our customers love using these resources to
supplement the bookwork. You can use the resources as you see fit for extra practice, to illustrate a concept
better, and even just for some fun. Enjoy!

https://l.mathmammoth.com/blue/percent

6 Percent (Blue Series)


Percent
Percent (or per cent) means per hundred or “divided by a hundred.”
That is because the word “cent” means one hundred.
The symbol for percent is %.
When you divide by 100, you get one hundredth (1/100).
Therefore, 8% means 8 per 100, which is 8/100.
Similarly, 67% means 67 divided by 100, or 67/100.
Since percentages are just hundredths, we can very easily write them as fractions and as decimals.
63 9
63% = = 0.63 9% = = 0.09
100 100

1. Write the shaded part and the unshaded part as fractions, as decimals and as percentages.

a. shaded b. shaded

= ________ = _______ % = ________ = _______ %

unshaded unshaded

= ________ = _______ % = ________ = _______ %

2. Write as percentages, fractions and decimals.

28
a. 28% = = 0.28 b. 17% = = ________ c. _______% = = 0.89
100

5
d. 60% = = ________ e. _______% = = ________ f. _______% = = 0.08
100

3. Typically, seven out of every 100 babies born in the River Creek Hospital have a birth defect, most of them
minor defects.

a. What typical percentage of the babies have birth defects?

b. What typical percentage of the babies do not have birth defects?

c. About how many babies with birth defects would you expect to find in a group of 500 babies?

d. About how many babies with birth defects would you expect to find in a group of 1,300 babies?

7 Percent (Blue Series)


Other fractions as percentages

What part of the pencils are short? Let’s rewrite 2/5 with a denominator of 100
using the method for equivalent fractions:
· 20

2 40 Now we can write 40/100 as 40%.


=
5 100 So, 40% of the pencils are short.
Two out of five, or 2/5 of them are short.
· 20

4. Write what part of the pencils are short, both as a fraction and as a percentage. Use equivalent fractions.

a. = = _____% b. = = _____% c. = = _____%


100 100 100

5. Convert the fractions into equivalent fractions with a denominator of 100, and then write them as percentages.

4 11 8
a. = = ______% b. = = ______% c. = = ______%
10 100 20 100 10 100

3 6 4
d. = = ______% e. = = ______% f. = = ______%
20 100 25 100 5 100

6. Write what part of the rectangle is shaded and what part is not shaded, both as fractions and percentages.

a. b. c.

Shaded: = _____% Shaded: = _____% Shaded: = _____%

Not shaded: = _____% Not shaded: = _____% Not shaded: = _____%

8 Percent (Blue Series)


Percentages that are more than 100%
The image shows 1 whole and 55/100. As a mixed number,
we write 1 55/100. As a decimal, we write 1.55.
Since 55/100 is 55%, and one whole is 100%, the image shows 155%.

We can use percentages that are more than 100%. Just remember that 100% is 1, and 1% is 0.01.
200 308
200% = = 2 308% = = 3.08
100 100

7. Write as fractions, decimals and percentages.

a. = _______ = ______% b. = _______ = ______% c. = _______ = ______%


100 100 100

8. Write as percentages, fractions and decimals.

a. 105% = = ______ b. 457% = = ______ c. _____% = = 2.09

506 482
d. ____% = = ______ e. ____% = = _____ f. _____% = = 3.11
100 100

9. Write the fractions as percentages.

a. About 4/5 ( _________ %) of the population of the United States is 14 years old or older.

b. About 2/25 ( _________ %) of the world’s population lives in North America.

c. The continent of Africa covers about 1/5 ( _________ %) of the Earth’s total land mass.

10. There are two trees growing in Sandy’s front yard. The taller one is 5/4 as tall as the shorter one.
a. Write the second sentence using a percentage instead of a fraction.

b. If the shorter tree is 160 cm tall, how tall is the taller tree?

9 Percent (Blue Series)


Change any fraction to a percentage
0 1 4.2 8
To write 1/7 as a percentage, you can either: 7 )1 0 0.0 0
• Divide 1 by 7 using long division or a calculator. You will get - 7
a decimal number. Express that as a percentage. OR, 30
-28
• Find 1/7 of 100%; in other words divide 100 by 7. 20
Then your answer is already a percentage.
- 14
60
Dividing 100 by 7, we get 14.28...
-56
Rounded to the nearest whole percent, that is 14%.
4
How many percent would 2/7 be?
What about 5/7?

11. Write the fractions as percentages. Use long division. Round your answers to the nearest percent.

a. = ______% b. = ______% c. = ______%

12. Write the fractions as percentages. Round the answers to the nearest percent.

a. According to a 2020 estimate, about 1/20 ( _______ %) of the population of Guatemala


is 65 years old or older.

b. In that same year, about 13/100 ( ________ %) of the population of Australia was 65 years old or older.

c. The Indian Ocean covers approximately 7/50 ( ________ %) of the Earth’s surface.

d. About 3/5 ( _________ %) of the world’s population lives in Asia.

13. Write as a percentage. Round your answers to the nearest percent.


a. 8/7 b. 1 3/8

How many percent of


a. b. c.
each figure is colored?

10 Percent (Blue Series)


What Percentage…?
What percentage A choir has 22 women and 18 One pair of jeans costs $25 and another
of the height of men. Find what percentage costs $28. How many percent is the
a 4-m tree is of the choir’s members are price of the cheaper jeans of the price
a 1-m sapling? men. of the more expensive jeans?

Look carefully at the questions above. Notice that the problems don’t tell you the percentage; in other
words, there is no number in the problem written as x%. Instead, they ask you to find it!

Questions with “What percentage…?” or “How many percent…?”

Asking “What percentage?” or “How many percent?” is the same as asking “How many hundredth parts?”
We can solve these questions in a two-part process:
1. First find out the part that is being asked for as a fraction. The denominator will probably not be 100.
2. Convert that fraction to a decimal. Then you can easily convert the decimal to a percentage!

Example 1. A choir has 22 women and 18 men. Find what percentage of the choir’s members are men.
1. Find what part (fraction) of the choir’s members are men. That is 18/40, or 9/20.
2. Write 9/20 as a percentage. You can use equivalent fractions: 9/20 = 45/100 = 45%.

Example 2. One pair of jeans costs $25 and another costs $28.
How many percent is the price of the cheaper jeans of the price of the more expensive jeans?
1. Write what part (fraction) the cheaper price is of the more expensive price. The answer is 25/28.
2. Write 25/28 as a percentage. A calculator gives 25/28 = 0.8928…
Rounded to the nearest whole percent, that is 89%.

1. a. What percentage of the height of a 4-m tree is the height of a little 1-m sapling?

b. How many percent is $12 of $16?

2. Find how many percent the shorter object’s height is of the taller object’s height.

6m 8m 300 cm 120 cm 4m 5m

a. b. c.

11 Percent (Blue Series)


3. A 2-year old child measures 80 centimeters tall and weighs 11 kilograms.
A 10-year old child measures 130 cm (1.30 m) tall and weighs 44 kilograms.

a. How many percent is the smaller child’s


age of the older child’s age?

b. How many percent is the smaller child’s


height of the older child’s height?

c. What percentage is the smaller child’s


weight of the older child’s weight?

4. Write the approximate percentages into the sectors in the circle graphs. Think of fractions!

a. b. c.

5. The circle graph at the right gives the angle measure of each sector
of the circle. Find what percentage each sector is of the whole circle.
Lastly, write that percentage in the sector in the image. Remember, the
whole circle is 360°.
Hint: Think what percentage 117° is of 360°.

6. Nine hundred sixty people gathered at a medical conference. Of them, 450 were
doctors, 220 were nurses and the rest were researchers. Find what percentage of
the people were doctors, what percentage were nurses and what percentage were
researchers.

Draw or sketch a circle


graph to represent the
situation in exercise #6.

12 Percent (Blue Series)


Percentage of a Number (Mental Math)
100% of something means all of it. 1% of something means 1/100 of it.

Since one percent means “a hundredth part,” calculating a percentage of a quantity is the same thing as
finding a fractional part of it. So percentages are really fractions!

How much is 1% of 200 kg? This means how much is 1/100 of 200 kg? It is simply 2 kg.
To find 1% of something (1/100 of something), divide by 100.
Do you remember how to divide by 100 in your head? Just move the decimal point two places to the left.
For example, 1% of 540 is 5.4, and 1% of 8.30 is 0.083.

To find 2% of some quantity, first find 1% of it, and double that.


For example, let’s find 2% of $6. Since 1% of $6 is $0.06, then 2% of $6 is $0.12.

To find 10% of some quantity, divide by 10.


Why does that work? It is because 10% is 10/100, which equals 1/10. So 10% is 1/10 of the quantity!
For example, 10% of $780 is $78. And 10% of $6.50 is $0.65.
(To divide by 10 in your head, just move the decimal point one place to the left.)

Can you think of a way to find 20% of a number?


________________________________________________________________________________

1. Find 10% of these numbers.

a. 700 _______ b. 321 _______ c. 60 _______ d. 7 _______

2. Find 1% of these numbers.

a. 700 _______ b. 321 _______ c. 60 _______ d. 7 _______

3. One percent of Mother’s paycheck is $22. How much is her total paycheck?

4. Fill in the table. Use mental math.

percentage ↓ number → 1,200 80 29 9 5.7

1% of the number

2% of the number

10% of the number

20% of the number

13 Percent (Blue Series)


5. Fill in this guide for using mental math with percentages:

Mental Math and Percentage of a Number

1
50% is . To find 50% of a number, divide by _______. 50% of 244 is _________.
2
1
10% is . To find 10% of a number, divide by ________. 10% of 47 is __________.

1
1% is . To find 1% of a number, divide by ________. 1% of 530 is __________ .

To find 20%, 30%, 40%, 60%, 70%, 80%, or 90% of a number, 10% of 120 is __________.
• First find __________% of the number, and 30% of 120 is __________.
• then multiply by 2, 3, 4, 6, 7, 8, or 9.
60% of 120 is __________.

6. Find the percentages. Use mental math.

a. 10% of 60 kg __________ b. 10% of $14 __________ c. 10% of 5 m __________

20% of 60 kg __________ 30% of $14 __________ 40% of 5 m __________

d. 1% of $60 __________ e. 10% of 110 cm __________ f. 1% of $1,330 __________

4% of $60 __________ 70% of 110 cm __________ 3% of $1,330 __________

7. David pays a 20% income tax on his $2,100 salary.


a. How many dollars is the tax?

b. How much money does he have left after paying the tax?

c. What percentage of his salary does he have left?

8. Nancy pays 30% of her $3,100 salary in taxes. How


much money does she have left after paying the tax?

9. Identify the errors that these children made. Then find the correct answers.

a. Find 90% of $55. b. Find 6% of $1,400.


Peter’s solution: Patricia’s solution:
10% of $55 is $5.50 1% of $1,400 is $1.40.
So, I subtract 100% − $5.50 = $94.50 So, 6% is six times that, or $8.40.

14 Percent (Blue Series)


Some more mental math “tricks”

90% of a quantity 25% of a quantity


First find 10% of the quantity and then 25% is the same as 1/4. So, to find
subtract that from 100% of it. 25% of a quantity, divide it by 4.

12% of a quantity 75% of a quantity


First find 10% of it. Then find 1% of it, and use that 75% is 3/4. First find 1/4 of the
1% to find 2% of it. Then add the 10% and the 2%. quantity and multiply that by 3.

10. Find percentages of the quantities.

a. 50% of 26 cm ____________ b. 25% of 40 mm __________ c. 80% of 45 m ____________

d. 75% of $4.40 ____________ e. 90% of 1.2 m ____________ f. 25% of 120 kg ____________

11. Fill in the mental math method for finding 12% of $65.
10% of $65 is $__________. 1% of $65 is $___________. 2% of $65 is $___________.

Now, add to get 12% of $65: $___________ + $___________ = $___________

12. Fill in the mental math shortcut for finding 24% of 44 kg.
25% of 44 kg is __________ kg. 1% of 44 kg is ___________ kg.

Subtract __________ kg − __________ kg = __________ kg

13. From her cell phone bill, Hannah sees that of the 340 text messages
she sent last month, 15% were sent during the night at a cheaper rate.
How many messages did Hannah send at night? During the day?

14. A herd of 40 horses had some bay, some chestnut and some white
horses. Thirty percent of them are bay, and 45% are chestnut.
How many horses are white?

15. A college has 1,500 students, and 12% of them ride the bus.
Another 25% walk to the college.
How many students do not do either?

15 Percent (Blue Series)


Percentage of a Number: Using Decimals
You have learned that finding 1% of a number means Next, let’s write those percentages as decimals.
finding 1/100 of it. Similarly, finding 60% of a We get:
number means finding 60/100 (or 6/10) of it.
1% of 90 60% of $700
In these types of expressions, the word “of”
↓ OR ↓
translates into multiplication:
0.01 · 90 0.6 · $700
1% of 90 60% of $700
↓ OR ↓ This gives us another way to calculate a certain
percentage of a number (or a percentage of some
1% · 90 60% · $700 quantity):

To calculate a percentage of a number, you need to make TWO simple changes:


1. Change the percentage into a decimal.
2. Change the word “of” into multiplication.

Example 1. Find 70% of 80.


Making the two changes, we write this as 0.7 · 80.
(Remember, in decimal multiplication, you multiply just as if there were no decimal points, and the answer
will have as many decimal digits as the total number of decimal digits in all of the factors.)
So, when you multiply 0.7 · 80, think of multiplying 7 · 80 = 560. Since 0.7 has one decimal digit, and 80 has
none, the answer has one decimal digit. Thus, 0.7 · 80 = 56.0 or just 56.
You can also use common sense and estimation: 0.7 · 80 must be less than 80, yet more than 1/2 of 80, which
is 40. Since 7 · 8 = 56, you know that the answer must be 56—not 5.6 or 560.

Example 2. Find 3% of $4,000.


First, write this as 0.03 · $4,000. Next, multiply without decimal points: 3 · $4,000 = $12,000.
Lastly, put the decimal point so that the answer will have two decimal digits: $120.00.

Example 3. Find 23% of 5,500 km.


Write this as 0.23 · 5,500 km and use a calculator. The answer is 1,265 km. This makes sense, because 10% of
5,500 km is 550 km, and 20% of it is 1,100 km. Therefore, 1,265 km as 23% of 5,500 km is reasonable.

1. “Translate” the expressions into multiplications by a decimal. Solve, using mental math.

a. 20% of 70 b. 90% of 50 c. 80% of 400

_______ · _______ = _______ _______ · _______ = _______ _______ · _______ = _______

d. 60% of $8 e. 9% of 3,000 f. 7% of 40 L

_______ · _______ = _______ _______ · _______ = _______ _______ · _______ = _______

g. 150% of 44 kg h. 200% of 56 students i. 2% of 1,500 km

_______ · _______ = _______ _______ · _______ = _______ _______ · _______ = _______

16 Percent (Blue Series)


2. “Translate” the other way: Write the multiplications as expressions of a “percentage of the number”.

a. 0.6 · 50 b. 0.03 · $400 c. 0.8 · 400 km

______% of _______ = _______ ______% of _______ = _______ ______% of _______ = _______

d. 0.08 · 6 e. 0.11 · $300 f. 0.2 · 70 kg

______% of _______ = _______ ______% of _______ = _______ ______% of _______ = _______

3. Use a calculator to find percentages of these quantities.

a. 17% of $4,500 b. 67% of 27 m c. 48% of 7.8 kg

4. Use mental math to find percentages of these quantities.

a. 25% of 240 m b. 80% of 30,000 km c. 75% of 3.2 kg

5. a. A lake has a 30-km long shoreline. Six percent of it is sandy beach.


What percentage of the shoreline is not sandy beach?

b. Find the length (in km) of the shoreline that is sandy beach.

6. Twenty percent of a university’s 4,000 students have a scholarship.

a. What percentage of the students do not have a scholarship?

b. How many students have a scholarship?

c. How many students do not have a scholarship?

7. A farmer had 1,200 hectares of land. He planted 30% of it


with wheat, 45% with corn and the rest with oats.
Find how many hectares he planted with each kind of grain.

17 Percent (Blue Series)


8. Identify the errors that these children made. Then find the correct answers.

a. Find 80% of 50. b. Find 75% of 84,000.


Gladys’s solution: Glenn’s solution:
80 · 50 = 4,000 This is the same as 84,000 ÷ 4 = 21,000.

9. Circle the expressions with the same value as 20% of $620.

0.02 · $620 $620 ÷ 5 $620 ÷ 10 · 2 2 · $62


1
· $620 0.2 · $620 20 · $620 $620 ÷ 4
5

10. About 69% of Italy’s population of 60 million live in cities.


About 37% of Tanzania’s population of 60 million live in cities.
How many more Italians than Tanzanians live in cities?

11. The table below shows Andy’s usage of time in one day.
a. Calculate the time he spent doing each activity. Round the minutes to the nearest minute.
b. Label the sections in the circle graph with the name of each activity.

Andy’s Usage of Time


Activity Percentage Minutes Hours/minutes
Sleep 38%
School 21%
Soccer 10% 144 2 h 24 min
Play 11%
Eating 9%
Chores 9%
Hygiene 2%
TOTAL 100% 1440 24 hours

18 Percent (Blue Series)


Discounts
Other than figuring sales tax, the area of life in which you will probably most often need to use
percentages is in calculating discounts.

A laptop that costs $600 is 20% off. What is the sale price?
Method 1. We calculate 20% of $600. That is the discounted amount in dollars.
Then we subtract that from the original price, $600.
20% of $600 is $120. And $600 − $120 = $480. So the sale price is $480.
Method 2. Since 20% of the price has been removed, 80% of the price is left.
By calculating 80% of the original price, you will get the new discounted price: 0.8 · $600 = $480

Two methods for calculating the discounted price:

1. Calculate the discount amount as a percentage 2. Find what percentage of the price is left. Then
of the original price. Then subtract. calculate that percentage of the normal price.

1. All of these items are on sale. Calculate the discount in dollars and the resulting sale price.

Price: $90 Price: $5 Price: $15


a. 20% off b. 40% off c. 30% off

Discount amount: $ 18 Discount amount: $__________ Discount amount: $__________


Sale price: $______________ Sale price: $______________ Sale price: $____________

2. A swimsuit that cost $25, was on sale for 20% off.


Monica tried to calculate the discounted price this way: $25 − $20 = $5.
What did she do wrong? Find the correct discounted price.

3. All these items are on sale. Find the discounted price.

a. Price: $1.20 b. Price: $18 c. Price: $150


25% off 25% off 30% off

Discount amount: $__________ Discount amount: $__________ Discount amount: $__________

Discounted price: $_________ Discounted price: $_________ Discounted price: $_________

d. Price: $20 e. Price: $4.00 f. Price: $1.30


40% off 10% off 50% off

Discount amount: $__________ Discount amount: $__________ Discount amount: $__________

Discounted price: $__________ Discounted price: $_________ Discounted price: $__________

19 Percent (Blue Series)


You can often use estimation when calculating the discounted price.
Example 1. A bicycle that costs $198.95 is discounted by 25%. What is the discounted price?
To estimate, round the original price of the bicycle to $200. Then, 25% of $200 is $50
(it is ¼ of it). So the discounted price is about $150.
Example 2. A laptop that costs $425.90 is discounted by 28%. What is the discounted price?
Round the discount percentage to 30%, and the price of the laptop to $430. 10% of $430 is $43.
30% of $430 is three times that much, or $129. Subtract using rounded numbers: $430 − $130 = $300.

4. Estimate the discounted price.


a. 30% off of a book that costs $39.90

b. 17% off of a sandwich that costs $12.50

c. 75% off of a swimming cap that costs $75.50

5. Which is a better deal? Estimate using rounded numbers and mental math.
a. 75% off of a brand-name mp3 player that costs $199
OR an equivalent off-brand mp3 player for $44.99.

b. 40% off of a new textbook that costs $89


OR a used copy, like new, of the same textbook for $39.90.

6. A company sells an app for $9.99. They estimate they would sell
50 copies of it in a week, with that price. If they discount
the price by 25%, they think they could sell 100 copies.
Estimate which way they would earn the most money.

Example 3. A pair of shoes costing $50 is discounted and now costs only $35. What is the discount
percentage?
Think about what fraction of the price “disappeared.” Then, write that fraction as a percentage.
We see that $15 of the price “went away.” The fraction of the price that was taken off is thus 15/50.
Now we simply rewrite 15/50 as 30/100, which is, as a percentage, 30%. So it was discounted by 30%.

7. Find the discount percentage.


a. Some jeans: original price, $50; discounted price, $45.

b. A phone: original price, $40; discounted price, $30.

c. A haircut: original price, $25; discounted price, $20.

8. Which of these methods work for calculating a discounted price of 25% off of $46?
$46 $46 $46 $46
0.25 · $46 0.75 · $46 $46 − $46 − ·3
25 4 4 4

20 Percent (Blue Series)


Practice with Percent
1. What is 70% of $380? 2. What percentage is $70 of $380?
Notice carefully the difference between the two questions above! Question #1 asks for a certain part (70/100)
of $380, and the answer will be in dollars. Question #2 asks for the percentage.
1. What is 70% of $380? 2. What percentage is $70 of $380?
First we find 10% of $380. That is just 1/10 of it, First we write what part $70 is of $380. It is
or $38. Then we multiply that by 7 to get 70% of simply 70/380 or 7/38. Writing this fraction
$380: 7 · $38 = $266. So 70% of $380 is $266. as a decimal, 7/38 = 0.184210526 ≈ 0.18,
which is 18%.
Alternatively, you could multiply 0.7 · $380 = $266.

If the percentage is known, and the total is known: If the percentage is unknown:
(What is x% of y?) (“What percentage?” / “How many percent?”)
1. Write the percentage as a decimal. 1. Write the part asked for as a fraction.
2. Multiply that decimal and the number. 2. Convert the fraction to a
decimal → percentage.
Or use mental math tricks for finding 1%, 10%,
20%, 30%, 25%, 50%, 75%, etc. of a number.

1. a. Find 10% of $50.

b. What percentage of $50 is $10?

2. a. Jenny ate 60% of a package of 25 cookies.


How many cookies did Jenny eat?

b. Jared ate 6 of the 25 cookies in the package.


What percentage of the cookies did Jared eat?

3. Use mental math to fill in the missing numbers:

a. Jack made 17 baskets out of 20 shots. b. Jack made baskets on 56% of 50 shots.

Jack made baskets on , or _______%, of his shots. Jack made ________ baskets in all.

c. Of 25 women surveyed, 60% like chocolate. d. 42 out of 200 citizens voted for Mr. X.

Of those 25, ______ women like chocolate. ______% of the citizens voted for Mr. X.

e. Of 1,000 boxes, 620 contained books. f. Out of 50 participants, 14% came late.

_________% of the boxes contained books. So, _________ participants came late.

21 Percent (Blue Series)


A diagram (bar model) can keep you from getting confused with fractions, percentages and actual quantities.
Example 1. 18 out of 25 swimmers in the club are practicing freestyle.
How many percent of the swimmers are practicing freestyle?
Each swimmer is one “block” in the diagram. The
whole diagram represents 25 swimmers, but it
also represents 100%. Therefore, one swimmer
(one block) represents 100% ÷ 25 = 4%.
So, 18 swimmers represent 18 · 4% = 72%.
You can also simply write the fraction 18/25 as a percentage: 18/25 = 72/100 = 72%.

4. Fifteen percent of the 40 workers in a shop


are over 50 years old. Now, think carefully.
Which of the following are correct?

a. 15/40 of the workers are over 50. OR 15/100 of the workers are over 50.

b. 6 workers are over 50. OR 15 workers are over 50.

In problems 5-7, you can draw a diagram to help you solve the problem.

5. Justin gave away 70% of the 20 stuffed animals he had.


How many does he have left?

6. Jerry painted 15 meters of fence out of the 25 meters that need to be painted.
What percentage of the job did he do?
(Round your answer to a whole percent.)

7. Marie wasted 30% of her $20 on candy.


How much money does she have left?

8. How many percent of each figure is colored?

a. b. c.

22 Percent (Blue Series)


Example 2. How many percent of a 5-m tree is a tiny 28-cm sapling?
1. Find out what part (what fraction) of a 5-m tree a 28-cm sapling is. We cannot
compare the two unless we write the heights in the same units. Five meters is 500 cm.
So, the 28-centimeter sapling is a 28/500 = 7/125 part of the 5-m tree.
2. Write 7/125 as a percentage. Using a calculator, 7 ÷ 125 = 0.056, or rounded
to two decimals, 0.06. This is a decimal. As a percentage, 0.06 is 6%.

9. a. Evelyn is 1 m 4 cm tall and Mary is 1 m 57 cm tall.


What percent of Mary’s height is Evelyn’s height?

b. Jacqueline is 90% of the height of her father, who is 1 m 90 cm tall.


How tall is Jacqueline?

10. Two college students, Peter and Jake, share a room. The rent is $450 per month, and they
share the rent equally. Peter earns $900 a month, and Jake earns $1,350.

a. Without calculating, determine which boy has to


use a bigger part of his earnings to pay the rent.
Explain your reasoning.

b. Now find what percentage of his earnings each boy uses for the rent.

11. a. Without calculating, determine which is more money: 11% of $402 or 12% of $298?
Explain how you figured it out.

b. Estimate the approximate difference between the two amounts.

12. The Roberts family drove a 1,920-kilometer trip in four days. On the
first day they drove 544 km, on the second day 448 km, on the third
day 640 km, and on the fourth day the rest of the way.

a. Find the average distance that they traveled per day.

b. For each day, find what percentage of the total trip they drove.

c. Suppose the family had divided the trip into four equal portions
and driven the same distance each day. What percentage of the
total trip would they have driven each day?

23 Percent (Blue Series)


Finding the Total When the Percentage Is Known
Use a bar model to find the unknown total when you know the percentage and the quantity.

Example 1. If 32 red marbles make up 4/5 of the total number of marbles, how many marbles are
there in all?
Look at the bar model. We have drawn the marbles as
divided into 5 equal “blocks.” Four of those five blocks
make up a total of 32 marbles. So, one block, or 1/5 of
the marbles, is 8 marbles. From that it is easy to calculate the total: 5 · 8 = 40 marbles.

The same reasoning works if the part of the marbles is given as a percentage instead of as a fraction:
Example 2. If 91 red marbles is 35% of the total number of marbles, how many marbles are there in all?
In the model, we need 100 little “blocks” with 35 of them colored (since 35/100 of the marbles are red.)

The calculation is done the same way: If 35 “blocks” or 35% make up 91 marbles, then one “block”, or one
percent, is 91 ÷ 35 = 2.6. Then, to find the total, simply multiply that number by 100: 2.6 · 100 = 260.

1. Margie gave away 40 marbles, which was 20% of the marbles that she had.
How many marbles did Margie have at first?
Hint: Instead of 100 blocks, you can use 5 blocks, each representing 20% or 1/5.

2. Emma cut down the amount of sugar in a recipe by 75%.


Now, she uses only 1/2 cup of sugar.
How much sugar did the recipe call for originally?
Hint: Instead of 100 blocks, you can use 4 blocks, each representing 25%.

3. When Eric bought a guitar for $90, he used up 12% of the money he had.
How much money did he have at first?

24 Percent (Blue Series)


Example 3. A phone was discounted by 40% and now costs $72. What was the price before the discount?
The cost now, $72, represents 60% of the original total—not 40%.
We can find 10% of the original price by dividing $72 ÷ 6 = $12. And from that, 100% of the price is
10 times that, or $120. If this confuses you, draw a bar model with 10 parts, each representing 10% of
the original price.

4. A dress was discounted by 20%.


The discounted price is $24.
What was the price before the discount?

5. A concert ticket was discounted by 60%.


The discounted price is $21.60.
What was the original price?

6. Joe spent 72% of his money, and now he has $56 left.
How much did Joe have to begin with?

7. Crystal spent 52% of her money and now she has $120 left.
How much did she spend?

8. Uncle Jack raises two different breeds of cows on his farm. Of his cows, 28% are Black Angus
and the rest are Hereford. If he has 420 Black Angus cows, how many Herefords does he have?

9. A survey found out that 16% of the people who had bought a certain brand
of coffee grinder were unhappy with it. If there were 126 people who were
happy with it, then how many people in total had bought that brand?

One calculator is discounted by 30% and now costs $42.


Another is discounted by 25% and now it also costs $42.
Which calculator had the cheaper original price? How much cheaper?

25 Percent (Blue Series)


Review: Percent
1. Find a percentage of a number 2. A fractional part as a percentage

What is 60% of 300 kilometers? What percent is 600 g of 2 kg?


Calculate 0.6 · 300 kilometers = 180 kilometers. 600 g 6 30
Write the fraction = = = 30%.
Or, using mental math, first calculate 10% of 2,000 g 20 100
300 kilometers, which is 1/10 of it, or 30 kilometers.
Then multiply 6 · 30 kilometers = 180 kilometers. One backpack costs $18 and another costs $29. What
percentage is the price of the cheaper backpack of the
Of the 15,400 workers in a city, 22% work in price of the more expensive one?
a steel factory. How many workers is that?
$18
Calculate: 0.22 · 15,400 = 3,388 workers. Write the fraction = 0.6206... ≈ 62%.
$29

1. Change the percentage into a decimal. 1. First write the fraction. Note that the two quantities
2. Then multiply the number by that decimal. in the fraction must both be in the same units: both
grams, both meters, both dollars, etc.
Alternatively, use mental math shortcuts
for finding 5%, 10%, 20%, 25%, 50%, etc. 2. Then convert the fraction into a decimal and
of a number. finally a percentage.

1. Write as percentages, fractions and decimals.

68
a. ______% = = _______ b. 7% = = _______ c. _______% = = 0.15
100

224
d. 120% = = _______ e. _____% = = ______ f. _______% = = 0.06
100

2. Fill in the table. Use mental math.

percentage ↓ number → 6 100 90 57 6

1% of the number

4% of the number

10% of the number

30% of the number

3. A group of skaters consists of 15 girls and 5 boys.


What percentage of the skaters are girls?

26 Percent (Blue Series)


4. Write as percentages. You may need long division in some problems.
If necessary, round your answers to the nearest percent.
a. 3/4

b. 2/25

c. 1 5/8

5. Emma is 1 m 63 cm tall and Madison is 1 m 22 cm tall. How


many percent is Emma’s height of Madison’s height?

6. A cheap chair costs $25. The price of another chair is 140% of that.
How much does the other chair cost?

7. A bag has 25 green marbles and some white ones, too. The green marbles are 20% of the total.
How many marbles are there in total? How many white marbles are there?

8. Andrew earns $2,000 monthly. He pays $540 of his salary in taxes.


What percentage of his income does Andrew pay in taxes?

9. Which is cheaper, an $18 shirt discounted by 20%,


or a $16 shirt discounted by 10%?

10. (Challenge) One square has sides 2 cm long, and another has sides 4 cm long.
How many percent is the area of the smaller square of the area of the larger square?

27 Percent (Blue Series)


Percentage Basics
Percent (or per cent) means per hundred or “divided by a hundred.”
(The word “cent” means one hundred.) So, simply put, percent means
a hundredth part.
To convert percentages into fractions, simply read the “per cent” as
“per 100.” Thinking of hundredths, you can also easily write them
as decimals.
Therefore, 8% = 8 per cent = 8 per 100 = 8/100 = 0.08.
Similarly, 167% = 167 per 100 = 167/100 = 1.67.

1. Write as percentages, fractions, and decimals.

59
a. 52% = = ________ b. _______% = = 0.07 c. _______% = = ________
100

382
d. 109% = = ________ e. _______% = = ________ f. 200% = = _______
100

A decimal number with two decimal digits is in hundredths, so it can easily


be written as a percentage. For example, 0.56 = 56%. But even if we have 91
3 or more decimals, we can still convert into percent.
0.091 = 9.1% =
1000
Example 1. The number 0.564 is 564 thousandths. As a percentage,
0.564 = 56.4%. Compare this to 0.56 = 56%. The decimal digit “4”that 387
0.387 = 38.7% =
follows the digits “56” is in the thousandths place, so it becomes 4 tenths 1000
of a percent (56.4%).

This is how to convert percentages with even more decimal digits:

decimal percentage decimal percentage


0.38429 = 38.429% 3.0281930 = 302.81930%

Think of it this way. Since 0.38 = 38%, any decimal digits that we have beyond 0.38 (the digits 429) simply
become decimal digits for the percentage. In effect, we move the decimal point two places to the right.

2. Write as percentages, fractions, and decimals.

a. 28.2% = = ________ b. 6.7% = = _______ c. ______% = = 0.891

1039 3409
d. 0.9% = = _______ e. ______% = = _______ f. ______% = = _______
10000 1000

g. 45.39% = 0.____________ h. 2.391% = 0.____________ h. ___________% = 0.942834

28 Percent (Blue Series)


Writing fractions as percentages

Sometimes you can easily convert a fraction For most fractions, you need to divide
to an equivalent fraction with a denominator in order to convert them into decimals 0.8 8 8 8
of 100. After that it’s easy to write it as first and then into percentages. 9 )8.0 0 0 0
a decimal and as a percentage. -7 2
Simply treat the fraction line as a
·4
80
division symbol and divide. You will
get a decimal. Then write the decimal
-72
46 184 as a percentage. 80
= = 1.84 = 184% - 72
25 100 80
Example 1.
-72
·4 8
= 0.888... ≈ 0.889 = 88.9% 8
9

3. Write the fractions as percentages.

8 142 24
a. = = _____% b. = = _____% c. = = _____%
25 100 200 100 20 100

4. Write as percentages. Use long division. Round your answers to the nearest tenth of a percent.

a. 11/8 b. 11/24

5. Use a calculator to convert the fractions into decimals. Round the decimals to four decimal digits. Then write
the decimals as percentages.

2 6
a. ≈ 0.6667 = 66.67 % b. ≈ ________ = ________%
3 7

17 52
c. ≈ ________ = ________% d. ≈ ________ = ________%
23 98

29 Percent (Blue Series)


If you are asked the percentage: Example 3. What percentage is 14 km of 75 km?
Asking what percent(age) is essentially the same 1. Write the fraction part/total: it is 14/75.
as asking “what part” or “what fraction.”
2. Then use a calculator and write it as a
part decimal. 14/75 = 0.186. Now write the decimal
1. Simply write the fraction .
total as a percentage: 0.186 = 18.6%.
2. Convert the fraction into a decimal, Normally, we round the result and say that 14 km
and then into a percentage. is about 19% of 75 km.

6. The circle graph shows the areas of the world’s


five oceans in square kilometers. The total area
of these oceans is 335,258,000 km2. To the
nearest tenth of a percent, find how many percent
each ocean is of the total area of the oceans.

Ocean Percentage of total area


Pacific Ocean
Atlantic Ocean
Indian Ocean
Southern Ocean
Arctic Ocean

7. The Carters live on a rectangular piece of land that measures 40 m × 35 m. The Joneses live on
a rectangular piece of land that measures 42 m × 39 m. To the nearest tenth of a percent, find
what percentage the area of the Carters’ land is of the area of the Joneses’ land.

8. Harry has two roosters, named Captain and Chief. The weight of Captain is 7/5 of the weight of Chief.
a. Write the second sentence above using a percentage instead of a fraction.

b. If Chief weighs 6 lb, how much does Captain weigh?

What percentage of
a. b.
each figure is colored?

30 Percent (Blue Series)


Solving Basic Percentage Problems
If the percentage is known and the total is known: If you are asked the percentage:
(What is x% of y?)
Asking “what percentage” is essentially the same
1. Write the percentage as a decimal. as asking “what part” or “what fraction.”
2. Multiply that decimal by the total. part
1. Write the fraction .
Or use mental math tricks for finding 1%, 10%, total
20%, 30%, 25%, 50%, 75%, etc. of a number. 2. Convert that fraction into a decimal,
and then into a percentage.

Example 1. A shirt that cost $34 was discounted by $4. What is the percentage it was discounted?
part $4
We write the fraction and get = 2/17 = 0.1176471 ≈ 11.8%.
total $34

Example 2. Find 59.2% of $2,600.


Write 59.2% as 0.592, and translate the word “of” into multiplication. We get 0.592 · $2,600 = $1,539.20.

Example 3. A meal that cost $14 was increased by 20%. What is the new price?
Because the price is increasing, the new price isn’t 20% but 100% + 20% = 120% of the original one.
So we can rewrite the previous sentence as: The new price = 1.2 · $14 = $16.80
In this case, since the numbers are easy, we could also use mental math. Ten percent of $14 is $1.40,
so the 20% increase is $2.80. The sum of the original price and the increase gives the new price:
$14 + $2.80 = $16.80.

1. Change the percentages into decimals.

a. 107% b. 16.67% c. 4.5%

2. Calculate the new, increased prices. Write the percentages as decimals and use multiplication.

a. Laptop: Original price $249.90, increase 6%.


New price = _________ · $249.90 = _______________

b. Biology textbook: Original price $82.40, increase 2.5%.


New price = _________ · $82.40 = _______________

c. Lunch buffet: Original price $18.50, increase 11.2%.


New price = _________ · $18.50 = _______________

3. Julia paid $325.08 of her $1,890 paycheck in taxes.


What percentage of her paycheck did she pay in taxes?

31 Percent (Blue Series)


4. Fill in Gloria’s solution to the following problem.
Roller blades originally cost $45.50, but now they are discounted by 13%. What is the new price?

Since 13% of the price is removed, _________% of the price is left. I write that percentage as
a decimal and multiply the original price by it: _________· $45.50 = _____________.

5. Calculate the discounted prices. Write the percentages as decimals and use multiplication.

a. Subscription to a magazine: Original price $78, discount 38%.

new price = _________ · $78 = _______________

b. Swimming goggles: Original price $14.95, discount 22.5%.

new price = _________ · $14.95 = _______________

6. The two rectangles are similar.

a. In what ratio are the corresponding sides of the rectangles?

b. In what ratio are their areas?

c. Calculate what percentage the area of the smaller rectangle


is of the area of the larger rectangle.

Sales Tax and Mental Math


A ticket to a circus costs $25. The sales tax is 6%. What is the final price you have to pay?
The sales tax is always added to the base price (the price without tax).
We simply calculate 6% of $25 and add that amount to $25.
Calculate in your head: 1% of $25 is $0.25. Six times that is $1.50. So the final price is $26.50.

7. Find the final price when the base price and sales tax rate are given. This is a mental math workout, so
do not use a calculator!

a. Bicycle: $100; 7% sales tax. b. Fridge: $400; 6% sales tax. c. Haircut: $50; 3% sales tax.

Tax to add: $_____________ Tax to add: $_____________ Tax to add: $_____________

Price after tax: $___________ Price after tax: $___________ Price after tax: $___________

32 Percent (Blue Series)


8. Here is another mental math workout! Once again, do not use a calculator. The sales tax is 5%. For each set
of items find the price with tax. Hint: To find 5% of a number, first find 10% and take half of that.

Price: $2 Price: $13 Price: $180 Price: $70 Price: $2 Price: $18

a. A microscope. b. Two shirts. c. Crayons and a toy horse.

d. A bag and a shirt. e. Six bottles of water. f. Two shirts and a bag.

9. Find the final price of a music CD with a base price of $11.50 when the sales tax is 6.7%.

10. Jeremy gets a 37.5% discount on a vacation package that normally costs $850.
Find what Jeremy will pay for the vacation package.

11. Patrick bought 5,000 m2 of land for a base price of $200,000. A 1.2% sales tax was added.
a. Find the total price Patrick paid.

b. Later, Patrick decides to sell 2,000 m2 of the land to a neighbor. What should Patrick
charge his neighbor in order to break even on what he paid for the part he’s selling?

12. a. The base price of a music CD is $12.50. It is first discounted by 20% and then a 7% sales tax is
added. What is the final price of the CD?

b. The base price of a pair of jeans is $55.97. They are first discounted by 40% and then a 5% sales
tax is added. What is the final price of the jeans?

13. Roger compared the unit prices of four different kinds of pasta.
One kind cost $2, another $1.50, another $2.20, and another $1.70.

a. Find the average price of the four types of pasta.

b. If each type of pasta were discounted by 10%, then what would the average price be?

33 Percent (Blue Series)


Percent Equations
Example 1. A handbag has been discounted by 23%, so now it costs $43.81. What was its original price?

Solution with an equation: To solve the equation, simply


divide both sides by 0.77:
Let p be the original price. A discount of 23% means that 23% of the
price (or 0.23p) is taken away from the price (p). As an expression, 0.77p = $43.81 ÷ 0.77
the discounted price is therefore p − 0.23p, which simplifies to 0.77p.
p ≈ $56.90
You can also reason that, after the discount, 77% of the price is left,
so the discounted price is 0.77p. Check:
Since the discounted price is $43.81, the equation to solve is 0.77 · $56.90 $43.81
0.77 · p = $43.81 43.813 ≈ $43.81

Solution with logical (proportional) reasoning: 77% —— $43.81

Again, we start out by reasoning that 77% of the price is left. In the 1% —— $43.81/77
chart on the right, the long lines mean “corresponds to” (not “equals”).
100% —— $43.81/77 · 100

100% —— $56.90

1. Write an expression for the final price using a decimal for the percentage.

a. Headphones: price $12, discounted by 24%. New price = _______________

b. A bag of dog food: price p, discounted by 11%. New price = _______________

c. Pizza sauce: price x, discounted by 17%. New price = _______________

d. Sunglasses: price s, price increased by 6%. New price = _______________

2. A computer is discounted by 25%, and now it costs $576. Let p be its price
before the discount. Select the equation that matches the statement above p + 0.25p = 576
and solve it.
p − 0.25p = 576

0.25p = 576

34 Percent (Blue Series)


3. The rent was increased by 5% and is now $215.25. What was the rent before the increase?
Write an equation for this situation and solve it.

4. A tablet is discounted by 30%. Matthew bought two of them, and he paid $98.
Find the price of the tablet before the discount (p). 2(p − 30) = 98

a. Find the equation on the right that matches the problem. 2p − 30 = 98

b. Solve it. 0.7p = 98

2(p − 0.3p) = 98

2(p − 0.3) = 98

5. a. Write an expression for the final price of a property with a base price of p when a 6.5%
sales tax and an 0.85% property tax are added to the base price.

b. Let’s say that the final price of the property in this situation is $16,639.25.
How much is the base price without the taxes?

35 Percent (Blue Series)


Percent proportion
Since percentages are fractions, we can easily write proportions to solve percent problems.
The basic idea is to write the fraction part/total and set that equal to the percentage, which
is written as a fraction with a denominator of 100.
part percent
That way we get this proportion: = .
total 100
Let’s solve the problem from the beginning of this lesson using a percent proportion.

Example 2. A handbag has been discounted by 23%, so now $43.81 77


it costs $43.81. What was its original price? =
p 100
The discounted price of $43.18 is the “part” and the original price
is the “total” (and is unknown). So we get the fraction $43.81/p. 77p = 100 · $43.81
The percentage 77% is written as the fraction 77/100. We get
the proportion: 77p = $4,381

$43.81 77 77p $4,381


= =
p 100 77 77
Its solution is on the left. p = $56.90

Example 3. Calculate 45% of 0.94 liters. Example 4. The other way to


calculate 45% of 0.94 liters is to
The total is 0.94 liters, and the part is unknown. We get the use decimal multiplication: it is
percent proportion 0.45 · 0.94 L = 0.423 L.
x 45 (Simply converting percentages into
=
0.94 L 100 decimals is often more efficient than
setting up a percent proportion.)
Solve it, and verify that you get x = 0.423 liters.

6. A fan is discounted by 22%, and now it costs $28. Let p be its price before the discount.
28/p = 78/100
a. Find the proportion on the right that matches the problem.
p/28 = 78/100
b. Solve it.
p/78 = 28

36 Percent (Blue Series)


part percent
7. Write and solve a percent proportion (according to the data below) in the form = .
total 100

a. How much is 56% of 4,500 km?

b. Thirty-eight percent of a number is 6.08. What is the number?

8. Alice bought 5 bottles of hair conditioner when the store had it at 15% off. Her total bill was $50.79.
What was the price of one bottle of hair conditioner before the discount?

37 Percent (Blue Series)


9. Matthew has to pay an annual property tax that is 0.8% of the accessed value (the official value
for tax purposes) of his land. The tax agency told him the tax is $95.20. From that information,
Matthew calculated the accessed value. What is the accessed value?

10. The price of electricity was lowered by 5%, so now it is $0.133 per kilowatt-hour.
What was the price before the decrease?

11. A store owner was planning for a big 30% off sale. However, she was rather unethical about it, and the
night before the sale, she increased the prices on some of the sale items. For example, she increased the
price by 30% for a roll of ribbon that did cost $5. What will the sale price of this roll of ribbon be?

12. The area of a triangular piece of land is 6 square kilometers. If the dimensions, including the base and
the altitude, of the triangle were increased by 10%, by how many percent would its area increase?
Hint: Make up a triangle with the given area. In other words, make up a base and an altitude so that the area is 6 km2.

A family’s water bill for the whole year was $584. From August through
December, the bill was 10% higher than from January through July because
of a 10% price increase. What was the monthly water bill before the
increase?

38 Percent (Blue Series)


Circle Graphs
A circle graph shows visually how a total is divided into parts (percentages).
Each of the parts (pie slices) is a sector, and each sector has a central angle.
To make a circle graph, we need to calculate the measure of the central angle
of each sector. For example, if a circle graph is supposed to show the percentages
25%, 13%, and 62%, simply calculate those percentages of 360° (the full circle):
25% of the total corresponds to 0.25 · 360° = 90°.
13% of the total corresponds to 0.13 · 360° = 46.8°.
62% of the total corresponds to 0.62 · 360° = 223.2°.

1. Sketch a circle graph that shows...

a. 50%, 25%, and 25% b. 33.3%, 33.3%, 1/6, and 1/6 c. 20%, 20%, 10%, and 50%

2. The table shows different kinds of specialty breads that a


grocery store ordered. Fill in the table. Make a circle graph.
(Note: You will need a protractor to draw the angles.)

Type Quantity Percentage Central Angle


white bread 50
bran bread 25
rye bread 30
corn bread 40
4-grain bread 55
TOTALS 200 100% 360°

3. a. Make a bar graph of the


quantities of each type of
bread from the table above. →

b. Does the bar graph


show percentages?

39 Percent (Blue Series)


4. Think of fractions. Estimate how many percent the sectors of the circle graphs represent.

a. b. c.

5. The table lists by flavor how many units of protein powder a company sold. Draw a
circle graph showing the percentages. You will need a protractor and a calculator.

Amount Percentage Central


Flavor
sold of total Angle
chocolate 67
vanilla 34
strawberry 16
blueberry 26
TOTALS 100% 360°

6. Mark polled some seventh graders about their favorite hobbies. Below are his results. Draw a circle
graph to show the percentages. Round the angles to whole degrees. You will need a protractor and
a calculator.

Favorite hobby Percentage Central Angle


Reading 12.3%
Watching TV 24.5%
Computer games 21%
Sports 22.3%
Pets 7.1%
Collecting 8.1%
no hobby 4.7%
TOTALS 100% 360°

40 Percent (Blue Series)


Percentage of Change
Percent(age of) change is a way to describe how much a price or some other quantity is increasing or
decreasing (changing). Let’s look at how to calculate the percentage a quantity is changing.

Example 1. A phone used to cost $50. Now it has been discounted to $45. What percentage was the
discount?
part
Since this problem is asking for the percentage, we will use our basic formula = percentage.
total
Because the change is relative to the original price, that original price becomes the “total” in our equation.
The “part” is the actual amount by which the quantity changes, in this case $5. So we get:
part $5
percentage = = = 1/10 = 10%
total $50
Essentially, we wrote what fraction the $5 discount is of the original $50 price and converted that fraction
into a percentage.

In summary: To calculate the percentage of change, use the same basic formula that defines a percentage:
part/total. Since the change is relative to the original price, the original price is the “total,” and the change in
price is the “part.”

part difference
percentage of change = =
total original

1. Write an equation and calculate the percentage of change.

a. A toy construction set costs $12. b. A sewing kit costs $20. It is discounted and
It is now discounted and costs only $8. costs only $16 now. What percentage is
What percentage is the discount? the discount?
difference
=
original

c. A bouquet of flowers used to cost $15, d. The price of a stove was $160. The price has
but now it costs $20. increased, and now it costs $200. What is the
What is the percentage of increase? percentage of increase?

41 Percent (Blue Series)


Compare these two problems:

Gasoline cost $3/gallon last week. Now it has Gasoline cost $3/gallon last week. Now it costs
gone up by 5%. What is the new price? $3.15. What was the percentage of increase?

1. Calculate 5% of $3. Since 10% of $3 is $0.30, we 1. Find how much was added to $3 to get $3.15
know that 5% is half of that, or $0.15. (the difference). That is $0.15.
2. Add $3 + $0.15 = $3.15/gallon. 2. Find what percentage $0.15 is of the original
That is the new price. price, $3. It is 15/300 = 5/100 = 5%. So the
percentage of increase was 5%.

To find the percentage of increase (in the right box above), we work “backwards” compared to when we find
the new price when the percentage of increase is known (in the left box above).

2. Solve and compare the two problems.

a. A shirt used to cost $24 but it was discounted b. A shirt used to cost $24. Now it is discounted
by 25%. What is the new price? to $18. What percentage was it discounted?

3. Solve and compare the two problems.

a. At 5 months of age, a baby weighed 5 kg. b. At 5 months, a baby weighed 6 kg. Over the next
At 6 months, the baby weighs 6 kg. month, his weight increased by 20%. What is his
What was the percentage of increase? weight at 6 months of age?

4. From June to July, the rent increased from $325 to $342. Then it increased again in August, to $349.
Which increase was a greater percentage?

42 Percent (Blue Series)


5. a. The price of a biology textbook was $60. Then it was lowered to $54.
Calculate the percentage change in price.

b. The price was increased back to $60. Calculate the percentage of increase.
Hopefully this is not a surprise to you, but the percentage is not the same as in part (a)!

6. A jacket cost $50. First its price was increased by 20%. Then it was discounted by 20%.
a. Calculate the final price. It will not be $50!

b. Since the original price was $50, use your answer from part (a) to calculate the true overall
percentage change in price.

7. Jake’s work hours were cut from 40 to 37.5 a week. Anita’s work hours were cut from 145 to
135 a month. Whose work hours were cut by a greater percentage?

8. The price of a vacuum cleaner that cost $100 is increased by 20%. Then it is increased by another 10%.
a. Find the price of the vacuum cleaner now.

b. Find the percentage of increase if the price had been increased from $100 to the final price
in one single increase. Note: the answer will not be 30%!

43 Percent (Blue Series)


Percentage of Change: Applications
Area

Example 1. A children’s playground measures 30 ft × 40 ft. It is enlarged so that each side is 10 ft longer.
What is the percentage of increase in the area?
The question doesn’t ask for the percent increase of each side, but of the area.
The original area is 30 ft × 40 ft = 1,200 sq. ft. The new area is 40 ft × 50 ft = 2,000 sq. ft.
Now we can find the percentage of increase. The fraction difference/original is (800 sq. ft. / 1,200 sq. ft) =
8/12. In lowest terms 8/12 becomes 2/3, which, as a percentage, is 66.7%.

Give your answers to the nearest tenth of a percent.

1. Find the percentage of increase in area when a 10 m × 10 m garden is enlarged to be 15 m × 15 m.

2. A newsletter has been printed on 21 cm × 29.7 cm paper. To save costs, it will be printed on
17.6 cm × 25 cm paper instead. By how many percent will the printable area decrease?

3. The sides of a square are increased by a scale factor of 1.15.

a. By what percentage does the length of each side increase?

b. What is the percentage of increase in area?


Hint: Make up a square using an easy number for the length of side.

c. (Challenge) Would your answers to (a) and (b) change if the shape were a rectangle?
A triangle?

44 Percent (Blue Series)


4. The graph below compares the production of water in California for usage in June, July,
and August of 2013 to the production in the corresponding months of 2014. Because of
imposed restrictions on water usage, less water was used in 2014 than in 2013.
Notice the scale is in millions of gallons. For example, in June, 2013, California produced 215,363 million
gallons of water, not 215,363 gallons.

Source: California Water boards

a. Why do you think the water production (and usage) is higher in July than in June?

b. Calculate the percentage of decrease between 2013 and 2014 for each of the three months.

c. In which month was the greatest percentage decrease in water production?

How can you see that from the graph?

5. The price of a jar of honey went from $5.50 to $6.00. Then it increased further to $6.50.
If the price were to increase by another $0.50 (from $6.50 to $7.00), would the percentage
increase be more than, less than, or the same as when the price was increased the first time?

45 Percent (Blue Series)


6. The sales tax is 7%. The price of a solar battery charger with tax is $69.99.

a. What is the price without tax?

b. Let’s say a merchant wants the price of the battery charger to be increased so that
the price including tax is $79.99. What percentage did the price including tax increase?

c. What is the percentage of increase of the price before tax?

7. A designer plans to use windows of the size 85 cm × 85 cm.


He changes his mind and uses windows that are both 10 cm wider and longer instead.

a. By how many percent does that change the area of one window?

b. Originally, the designer was going to use 20 of the smaller windows in the house.
About how many of the bigger windows cover the same area as 20 of the smaller ones?

8. a. Three items, with prices of $50, $60, and $70, have their prices increased by $10.
For which item is the percentage increase in price the greatest?

b. Three items, with prices of $50, $60, and $70, have their prices discounted by 12%.
Which item’s price decreases the most (in dollars)?

46 Percent (Blue Series)


9. The population of the state of Kentucky was 3,038,000 in 1960 and 3,219,000 in 1970.
Calculate the percentage of increase in the population of Kentucky during that decade.

10. The table shows the population of Kentucky every 10 years. The line graph shows the same
information. Your task is to calculate the percentage of increase in each decade. You already
did that for 1960-1970 in the previous exercise, so write that answer in the row for 1970.

% increase
Year Population
in the decade
1960 3,038,000 N/A
1970 3,219,000
1980 3,661,000
1990 3,685,000
2000 4,042,000
2010 4,340,000

11. The population of Kentucky grew more steeply in one of these decades than any of the others.
a. Which decade? From ___________ to ___________.

b. How can you tell which decade it is from the graph?

12. (Optional.) Make a line graph that shows the change in the population where you live (your town, city,
parish, county, state, province, country, etc.). Also calculate the percentage of increase or decrease for
each time period. You can often find population statistics in Wikipedia, for example. You can also search
the Internet (with adult supervision) for “your area population statistics,” where your area is the place that
you chose.

47 Percent (Blue Series)


Comparing Values Using Percentages
What percentage more/less/bigger/smaller/taller/shorter ...?

Example 1. A car weighs 2,000 kg. Another car weighs 2,500 kg. The second car is heavier, but how much
heavier than the first one is it?
To answer that question, we cannot just look at the difference of 500 kg and say that 500 kg is “a lot” or “a
little.” Instead, we need to take into account the sizes of the two items being compared and consider their
relative difference.

actual difference
The relative difference (or percentage of difference) of two values is the fraction .
reference value
This fraction is usually expressed as a percentage.

The problem is in determining which of the two values is the reference value. In this case, we want to find out
how much heavier the second car is than the first car, which means we use the weight of the first car as the
reference value. In another words, we are comparing the second car to the first car. It is as if the lighter car
were there first, and we are comparing a “newcomer” car to this first car.
500 kg 5 1
So the relative difference of the two weights is = = = 25%.
2,000 kg 20 4
This means the second car is 25% heavier than the first one. This gives us a precise value.
500 kg 5 1
Choosing the second car’s weight as the reference value gives: = = = 20%.
2,500 kg 25 5
This means the first car is 20% lighter than the second car.

Example 2. Southcreek College has 2,600 students and West River College has 2,400 students.
How many percent more students does Southcreek College have than West River College?
The difference in student count is 200. But which number is our reference? Since West River College is
mentioned after the word “than,” we are comparing Southcreek College to West River College. So the student
count of West River College is our reference number.
difference in student count 200 2 1
The fraction is = = = = 0.08333... ≈ 8.3%.
reference student count 2,400 24 12

So Southcreek College has approximately 8.3% more students than West River college.

1. Compare the taller object to the shorter one and calculate their relative difference.

6m 8m 300 cm 120 cm 4m 5m

difference
a. = b. c.
reference

48 Percent (Blue Series)


You may use the calculator in all problems in this lesson from this point on.

2. Erica is 140 cm tall, and Heather is 160 cm tall. Fill in the blanks.

Heather is = ______% taller than Erica. Erica is = ______% shorter than Heather.

3. a. Refer to the chart. Calculate the percentage Metropolitan Population


differences to the tenth of a percent. Country
area (millions)
The population of Tokyo is __________% larger Tokyo Japan 37.80
than the population of Delhi. Seoul South Korea 25.62
The population of Moscow is __________% smaller Shanghai China 24.75
than the population of Mexico City. Karachi Pakistan 23.50
Delhi India 21.75
b. Compare how much larger the population of Seoul is Mexico City Mexico 21.60
than the population of New York, and then again how Sao Paulo Brazil 21.20
much larger the population of New York is than the
population of Moscow. Jakarta Indonesia 20.00
New York City United States 19.95
Which is a larger percentage difference?
Mumbai India 20.75
Moscow Russia 15.51

When there is no clear way to choose a reference value when calculating the relative (percentage)
difference, you can use the average of the two values as a reference value.
Example 3. The monthly subscription fees to two math practice websites are $9.99 and $12.95.
What is their relative difference?
The average of the two prices is ($9.99 + $12.95)/2 = $11.47.
$12.95 − $9.99 2.96
The relative difference is therefore = ≈ 25.8%.
$11.47 11.47

4. The area of one park is 14,000 square feet, and the area of another park is 10,000 square feet.
Use their average area to calculate the relative percentage difference between their areas.

5. KeepCool company charges $28 per hour for labor, and CityFreez charges $32 per hour.
a. Use the average rate for labor to calculate the percentage difference in the rates.

b. Now compare the costs for two hours of labor. What is the percentage difference?

49 Percent (Blue Series)


Example 4. One cat weighs 1.2 kg and another weighs 1.5 kg.
Compare
Question 1. What percentage is the smaller cat’s weight of the bigger cat’s weight? these five
Solution: Write what fraction the smaller cat’s weight is relative to the bigger cat’s weight: types of
questions
smaller cat’s weight 1.2 kg 12 4 carefully!
It is = = = = 80%.
bigger cat’s weight 1.5 kg 15 5

Question 2. What percentage is the bigger cat’s weight of the smaller cat’s weight?
Solution: Write what fraction the bigger cat’s weight is relative to the smaller cat’s weight:
bigger cat’s weight 1.5 kg 15 5
It is = = = = 1 ¼ = 125%.
smaller cat’s weight 1.2 kg 12 4

Question 3. How much more (in percent) does the bigger cat weigh than the smaller cat?

Solution: This is a percentage difference relative to the smaller cat. Write the fraction (difference / reference):
difference in weight 0.3 kg 3 1
It is = = = = 25%.
smaller cat’s weight 1.2 kg 12 4

Question 4. Compare the weight of the smaller cat to the weight of the bigger cat.
Solution: This percentage difference is relative to the weight of the bigger cat:
difference in weight 0.3 kg 3 1
It is = = = = 20%.
bigger cat’s weight 1.5 kg 15 5

Question 5. What is the relative difference between the two cats’ weights?

Solution: No reference is specified, so let’s use the average weight: ½(1.5 kg + 1.2 kg) = 1.35 kg.
difference in weight 0.3 kg 30 2
It is = = = = 0.222... ≈ 22.2%.
average weight 1.35 kg 135 9

There are five different questions with five different solutions. Note the underlined key words!
You could be asked, “What percentage is (this) of (that)?”
OR: “What percentage more/less/smaller/bigger is (this) than (that)?”
Moreover, the order of comparison matters: The keywords “of,” “than,” and “to” mark the cat that we are
comparing to (the reference cat).

6. One bean plant is 12 cm tall, and another is 16 cm tall.


a. What percentage is the shorter plant’s b. How much taller (in percent) is the taller plant
height of the taller plant’s height? than the shorter plant?

c. Compare the height of the shorter plant to d. Find the relative difference in the heights
the height of the taller one. using their average height.

50 Percent (Blue Series)


7. Only one of the students gets the answer correct in each case. Who is it?
a. Jack made a tower of blocks 150 cm tall. Baby made a block tower that was 30 cm tall.
What percentage is the height of Baby’s tower of the height of Jack’s tower?

Elijah: Angela: Mary:


I write the fraction I write the fraction
I subtract 150 − 30 = 120%. 120 cm 12 4 30 cm 3 1
= = = 80% = = = 20%
150 cm 15 5 150 cm 15 5

b. The school orchestra has 26 boys and 14 girls. How many percent bigger is the number of boys than
the number of girls?

Elijah: Angela: Mary:


I subtract 26 − 14 = 12 and I write the fraction
write the fraction I subtract 26 − 14 = 12%. 14 7
12 6 = ≈ 54%
26 13
= ≈ 86%
14 7

8. Percentage comparisons can be misleading without the actual


Number of violent crimes in 2013-2014
data. To see that, consider the number of violent crimes
committed in two imaginary counties in 2013 and 2014.
County A County Z
a. Calculate the percentage of increase in violent crime
2013 2 454
for the two counties in the table.
2014 4 512
b. Let’s say you were thinking of moving into one of these two
counties, and you were told only the percentages of change Percentage
for violent crime. Which county might you move into? of increase

Yet, taking the actual data into account, your perception about these two areas would change a lot!

The table lists the number of domestic burglaries in three counties


in Wales for two different time periods.

Number of domestic burglaries


Ceredigion Conwy Gwynedd
April 2011 to March 2012 93 256 200
April 2012 to March 2013 56 229 138

Approximate percent of change

Determine without a calculator or paper and pencil (just figure in your head)
which county experienced the greatest percentage of decrease in domestic burglaries.

51 Percent (Blue Series)


Simple Interest
When you deposit money into a savings account, normally the bank actually pays you for the use of your
money. The amount you deposit is called the principal. The amount the bank pays is called the interest.
Similarly, if you borrow money, it is not free. You not only pay back the principal (the actual amount you
borrowed) but you also pay interest on the amount of money you borrowed.
Interest is normally defined as a certain percentage, called the interest rate, of the principal. For example, the
bank might charge you an interest rate of 7.9% on a loan. Often, people simply say “7.9% interest” instead of
“a 7.9% interest rate” or “an interest rate of 7.9%.”
In this lesson, we study only so-called simple interest, which means that the interest is added to the principal
only at the end of the time period during which the money is invested or borrowed. In real life, banks usually
calculate compound interest, which means that the interest is added to the principal at certain regular
intervals (such as every month or even every day) during the period of the loan or investment.

Example 1. How much interest does a principal of $2,000 earn in a year if the yearly interest rate is 5%?
The interest is simply 5% of $2,000, which is $100.

Example 2. You get a $3,000 loan with an annual interest rate of 8.5%.
You pay the loan back after 3 years. How much do you have to pay back?
The interest for one year is simply 8.5% of $3,000. For three years, it is three times that much.
Of course you also have to pay back the of principal $3,000. So the total amount you pay back is:
$3,000 + 3 · 0.085 · $3,000 = $3,000 + $765 = $3,765.

We can see from the above examples that to calculate the interest, we simply take the interest rate times
the principal times the time period. The formula for calculating simple interest (I) is usually given as:

I = Prt
where P is the principal, r is the interest rate, and t is the time.

You may use a calculator in all problems in this lesson.

1. Calculate the interest and the total amount to be paid back on these investments.
a. Principal $5,000; interest rate 3%; time 1 year
Interest: _________________________ Total to withdraw: ______________________

b. Principal $3,500; interest rate 4.3%; time 4 years


Interest: _________________________ Total to withdraw: ______________________

c. Principal $20,000; interest rate 7.6%; time 10 years


Interest: _________________________ Total to withdraw: ______________________

52 Percent (Blue Series)


2. Sandy plans to invest $3,000 for three years. She could either put it into a savings account with an annual
interest rate of 3.4% or get a Certificate of Deposit (CD) for 5 years with an annual interest rate of 3.92%.
However, if she withdraws the money from the CD before 5 years is up, the bank charges her a penalty of
6 months interest. Which allows Sandy to earn more money on her investment in 3 years?

Certificate of Deposit
Savings Account
Time period: 5 years
Interest rate: 3.4%
Interest rate: 3.92%

Example 3. Andy borrows $2,000 with an annual interest rate of 12.45%.


He is able to pay it back after 7 months. How much will Andy pay to the lender?
Notice that the interest rate is annual (yearly) but the time period is in months. Therefore:
(1) We need to either use a monthly interest rate. To get that, simply divide the annual interest rate by 12.
(2) Or we need to convert the time of 7 months into years. Of course, 7 months is simply 7/12 years.
Let’s use the first option. The monthly interest rate is 12.45% ÷ 12 = 1.0375%. Then, the interest is
0.010375 · 7 · $2,000 = $145.25. So Andy has to pay back $2,145.25.

3. Elizabeth bought a tablet for $450 on credit with a 12.9% annual interest rate.
a. How much interest (in dollars) will she pay in a month?

b. In a day?

53 Percent (Blue Series)


4. A credit card has a monthly interest rate of 1.09%, which doesn’t sound like much. How much interest
will you pay if you purchase a couch for $690 with the credit card and pay it back after two years?

5. Jerry took out a loan for $850 for 10 months with an annual interest rate of 10.8%. How much less interest
would he have paid if instead he had taken out a loan for 7 months with an annual interest rate of 9.5%?

6. John uses his credit card to finance a car for $26,000. The annual interest rate on his card is low, 2.75%,
but only for the first 12 months. After that, if Jon hasn’t paid the full amount back, the annual interest rate
jumps to 9.95%. Calculate how much John ends up paying back if he cannot pay the total during the first
12 months, but pays the entire amount after 2.5 years.

54 Percent (Blue Series)


Example 4. Eric borrowed $1,500 for 8 months and paid back $1,578. What was the interest rate?
The actual interest was $1,578 − $1,500 = $78. The loan was for 8 months, so the monthly interest
was $78 ÷ 8 = $9.75.
This amount is $9.75/$1,500 = 0.0065 = 0.65% of the principal. So the monthly interest rate was 0.65%.
The annual interest rate is 12 times that, or 7.8%.

Another solution. We could also write an equation,


using the formula I = Prt. $78
$1,500 · r · 8 months =
This formula is for the actual amount of interest (I), not the
total Eric paid back. The actual interest I was $78. We also $1,500 · r · 8 months $78
=
know that the time was 8 months and that the principal was $1500 $1500
$1,500. Substituting those values into the formula and
keeping the units, we get 78
r · 8 months =
1500
$78 = $1,500 · r · 8 months
78
It helps to write the equation so that the variable is on r · 8 months = 1500
the left (inverting the sides): 8 months 8 months
$1,500 · r · 8 months = $78 0.0065
r =
month
To solve this equation for r, we divide both sides by
$1,500 and then also by 8 months. See the solution r = 0.65% per month
on the right.

7. a. You borrowed $1,000 for one year. At the end of the year, you had to pay back $1,045.
What was the interest rate?

b. You borrowed $12,000 for five years. At the end of the five years you paid the bank back $15,600.
What was the interest rate?

55 Percent (Blue Series)


8. Alice opened a savings account that paid an interest rate of 6%. After ten years her account
contained $12,000. How much was the original principal?

9. How long would you have to invest $2,000 in order to earn $500 in interest, if the annual interest rate
is 11.5%?

10. What rate of interest do you need in order to earn $350 in interest in 2 years with a principal of $1,800?

56 Percent (Blue Series)


11. A family purchased a vacation package for $4,055 with a credit card that charged 11.95% annual interest.
When they paid off their credit card, they ended up paying the credit card company $4,741.48.
How long did it take for the family to pay off the debt for their vacation?

Let’s look at just one example of how compound interest is calculated!


Jayden buys a motorcycle for $5,000 with a credit card that has a 6%
annual interest rate. Compounding the interest means that the interest is
added to the principal at certain intervals, in this case each month.
Study the table below. Note that a 6% annual interest rate means 0.06/12 = 0.005 or 0.5% monthly
interest rate.
a. Fill in the table, calculating the interest for each month and adding it to the principal.
The following month, use the new principal to calculate the interest.

Month Monthly interest Principal

0 $5,000

1 $5,000 · 0.005 = $25 $5,025

2 $5,025 · 0.005 = $25.125 $5,050.125

3 $5,050.125 · 0.005 ≈ $25.251 $5,075.376

4 ____________ · 0.005 ≈

5 __________________ · 0.005 ≈

b. There is a formula for compound interest that allows these calculations to be done quicker. For this
situation, the total amount owed after month n is

$5,000 · 1.005n

For example, after 12 months, Jayden owes $5,000 · 1.00512 = $5,308.39. Use the formula to calculate
how much Jayden pays if he pays back the loan at the end of 2 years.

57 Percent (Blue Series)


Final Review
1. Find the percentage of the given quantity.

a. 9.2% of $150 b. 45.8% of 16 m c. 0.6% of 700 mi

2. All these items are on sale. Calculate the new, discounted prices.

a. b. c.

Price: $9 Price: $6 Price: $90


20% off 25% off 30% off

New price: $___________ New price: $____________ New price: $__________

3. A flashlight is discounted by 18%, and now it costs $23.37. Let p be its price
before the discount. Find the equation that matches the statement above p − 0.18 = $23.37
and solve it.
p − 18 = $23.37

0.82p = $23.37

0.18p = $23.37

4. Two brothers, Andy and Jack, shared the price of a new computer so that Andy paid
2/5 (or 40%) and Jack paid 3/5 (or 60%) of the price. The computer cost $459, and there
was a sales tax of 7% that was added onto the price. Calculate Andy’s and Jack’s shares.

5. A portable reading device costs $180. Now it is discounted and costs $153.
What was the percentage of discount?

58 Percent (Blue Series)


6. The two right triangles on the right are similar.

a. Calculate what percentage the area of the smaller triangle


is of the area of the larger triangle.

b. In what ratio are the corresponding sides of the triangles?

c. In what ratio are their areas?

7. A wall painting was planned to be 5 m by 3 m in size. If both of its sides are increased by 20%,
by what percentage will the area of the painting increase?

8. In a race, Old Paint finished in 120 seconds, and The Old Gray Mare finished in 200 seconds.

a. How many percent quicker was Old Paint than The Old Gray Mare?

b. How many percent slower was The Old Gray Mare than Old Paint?

c. What is the relative difference in their times?

9. Noah takes a $4,000 loan at a 7.8% annual interest rate to purchase a car. At the end of the first year, he
pays back $2,000 of the principal of the loan. At the end of the second year, he pays back the rest of the
principal and all of the interest. How much total interest does he have to pay? Assume simple interest,
which is calculated and paid only at the end of the period of the loan.

59 Percent (Blue Series)


60 Percent (Blue Series)
Math Mammoth Percent Answer Key
Percent, pp. 7-10
Page 7
1. a. shaded 67/100 = 0.67 = 67%; not shaded 33/100 = 0.33 = 33%
b. shaded 4/100 = 0.04 = 4%; not shaded 96/100 = 0.96 = 96%
2.
28 17 89
a. 28% = = 0.28 b. 17% = = 0.17 c. 89% = = 0.89
100 100 100
60 5 8
d. 60% = = 0.60 e. 5% = = 0.05 f. 8% = = 0.08
100 100 100

3. a. About 7% of the babies have birth defects.


b. About 93% percent of the babies do not have birth defects.
c. You could expect to find about 35 babies with birth defects in a group of 500 babies.
d. You could expect to find about 91 babies with birth defects in a group of 1,300 babies.
Page 8
4.
1 50 1 25 1 20
a. = = 50% b. = = 25% c. = = 20%
2 100 4 100 5 100

5.

4 40 11 55 8 80
a. = = 40% b. = = 55% c. = = 80%
10 100 20 100 10 100
3 15 6 24 4 80
d. = = 15% e. = = 24% f. = = 80%
20 100 25 100 5 100

6. a. shaded 3/5 = 60%; not shaded 2/5 = 40%


b. shaded 3/4 = 75%; not shaded 1/4 = 25%
c. shaded 8/10 = 80%; not shaded 2/10 = 20%
Page 9
7.
112 109 278
a. = 1.12 = 112% b. = 1.09 = 109% c. = 2.78 = 278%
100 100 100

8.

a. 105% 105 = b. 457% 457 = c. 209% 209


= 100 1.05 = 100 4.57 = 100 = 2.09
d. 506% 506 = e. 482% 482 = f. 311% 311
= 100 5.06 = 100 4.82 = 100 = 3.11

9. a. About 4/5 ( 80 %) of the United States population is 14 years old or older.


b. About 2/25 ( 8 %) of the world’s population lives in North America.
c. The continent of Africa covers about 1/5 ( 20 %) of the Earth’s total land mass.
10. a. The taller one is 125% as tall as the smaller one.
b. The smaller tree is 160 cm tall, so the taller tree is 200 cm tall.

61 Percent (Blue Series)


Percent, cont.
Page 10
11.
3 1 5
a. = 43% b. = 13% c. = 56%
7 8 9

2. a. About 1/20 ( 5% ) of the population of Guatemala is 65 years old or older. (2020 estimate)
b. About 13/100 ( 13% ) of the population of Australia is 65 years old or older. (2020 estimate)
c. The Indian Ocean covers approximately 7/50 ( 14% ) of the Earth’s surface.
d. About 3/5 ( 60% ) of the world’s population lives in Asia.
13. a. 114% b. 138%
Puzzle corner.
a. Each tiny white square is 1/64 of the whole. So the colored area as a fraction is (1/4) − (2/64) = 14/64 = 7/32.
As a percentage, that is about 22%.
b. Each tiny colored triangle is 1/16 of the whole. So, the colored area is 9/16.
As a percentage, this is about 56%.
c. Each tiny colored triangle is 1/16 of the whole. The colored area is 1/2 + 3/16 = 11/16.
As a percentage, this is about 69%.

What Percentage . . . ?, pp. 11-12


Page 11
1. a. 25% (1/4 = 25/100)
b. 75% (12/16 = 3/4 = 75/100)
2. a. 75% (6/8 = 3/4 = 75/100) b. 40% (120/300 = 12/30 = 2/5 = 40/100) c. 80% (4/5 = 80/100)
Page 12
3. a. 20% (2/10 = 20/100)
b. 62% (80/130 = 0.615384...)
c. 25% (11/44 = 1/4 = 25/100)
4.

a. b. c.

5. Divide each angle in degrees by 360.


For example, 66° out of 360° is 66/360 = 0.18333... = 18%.

Step → Given: Convert from Round to nearest


↓ Sector sector size (°) degrees (÷ 360°) whole %
1 (Grey) 15° 0.041666... 4%
2 (Brown) 32° 0.0888... 9%
3 (Yellow) 130° 0.36111... 36%
4 (Green) 66° 0.18333... 18%
5 (Violet) 117° 0.325 33%
TOTAL 360° 1 (circle) 100%

62 Percent (Blue Series)


What Percentage . . . ?, cont.
Page 12
6. Of the 960 people at the conference, 450/960 = 0.468... ≈ 47% were doctors,
220/960 = 0.229... ≈ 23% were nurses and
(960 − 450 − 220) / 960 = 290/960 = 0.302... ≈ 30% were researchers.
Puzzle Corner: To produce the image at the right (→), find:
47% of 360 degrees (169°),
30% of 360 degrees (108°) and
23% of 360 degrees (83°).
Then use those angles to draw the circle graph.

Percentage of a Number (Mental Math), pp. 13-15


Page 13

Can you think of a way to find 20% of a number?


Find 10% of the number and then double it.
OR Divide the original quantity by 5.

1. a. 70 b. 32.1 c. 6 d. 0.7
2. a. 7 b. 3.21 c. 0.6 d. 0.07
3. Her total paycheck is $2,200.
4.
percentage / number 1,200 80 29 9 5.7
1% of the number 12 0.8 0.29 0.09 0.057
2% of the number 24 1.6 0.58 0.18 0.114
10% of the number 120 8 2.9 0.9 0.57
20% of the number 240 16 5.8 1.8 1.14

Page 14
5.
Mental Math and Percentage of a Number
1
50% is . To find 50% of a number, divide by 2 . 50% of 244 is 122 .
2
1
10% is . To find 10% of a number, divide by 10 . 10% of 47 is 4.7 .
10
1
1% is . To find 1% of a number, divide by 100 . 1% of 530 is 5.3 .
100

To find 20%, 30%, 40%, 60%, 70%, 80%, or 90% of a number, 10% of 120 is 12 .
• First find 10% of the number and 30% of 120 is 36 .
• then multiply by 2, 3, 4, 6, 7, 8, or 9. 60% of 120 is 72 .

6.
a. 10% of 60 kg = 6 kg b. 10% of $14 = $1.40 c. 10% of 5 m = 0.5 m
20% of 60 kg = 12 kg 30% of $14 = $4.20 40% of 5 m = 2 m
d. 1% of $60 = $0.60 e. 10% of 110 cm = 11 cm f. 1% of $1,330 = $13.30
4% of $60 = $2.40 70% of 110 cm = 77 cm 3% of $1,330 = $39.90

63 Percent (Blue Series)


Percentage of a Number (Mental Math), cont.
Page 14
7. a. The tax would be $420.
b. He would have $1,680 left after taxes.
c. He has 80% left.
8. Nancy has $2,170 left after paying taxes.
9. a. Peter figured if he subtracted 10% from the total amount, he would have 90% left. However, Peter subtracted 10%
from the number 100 instead of from $55. The correct way to do it is to subtract $5.50 from $55. The correct answer
is $49.50.
b. Patricia moved her decimal point one place too many to the left. One percent of $1,400 is actually $14.
Multiply that by six to find the correct answer $84.
Page 15
10.
a. 50% of 26 cm = 13 cm b. 25% of 40 mm = 10 mm c. 80% of 45 m = 36 m
d. 75% of $4.40 = $3.30 e. 90% of 1.2 m = 1.08 m f. 25% of 120 kg = 30 kg

11. 10% of $65 is $6.50 . 1% of $65 is $0.65 . 2% of $65 is $1.30 .


Now, add to get 12% of $65: $6.50 + $1.30 = $7.80
12. 25% of 44 kg is 11 kg. 1% of 44 kg is 0.44 kg.
Subtract 11 kg − 0.44 kg = 10.56 kg
13. Hannah sent 51 messages during the night. She sent 289 messages during the day.
14. 100% − (30% + 45%) = 25% are white. Ten white horses is 25% of 40.
15. 945 students do not do either.
To figure it out, first calculate 12% of 1,500, then calculate 25% of 1,500, and subtract both amounts from 1,500.
Now, to find 12% of 1,500. First find 10% of 1,500, which is 150. 1% of 1,500 is 15, so 2% of 1,500 is double that
or 30. Totaling the parts, we find that 12% of 1,500 is 150 + 30 = 180. To find 25% of 1,500, we can divide
1,500 ÷ 4 = 375. Then we subtract 1,500 − 180 − 375 = 945.

Percentage of a Number: Using Decimals, pp. 16-18


Page 16
1.
a. 20% of 70 b. 90% of 50 c. 80% of 400
0.2 · 70 = 14 0.9 · 50 = 45 0.8 · 400 = 320
d. 60% of $8 e. 9% of 3,000 f. 7% of 40 L
0.6 · $8 = $4.80 0.09 · 3,000 = 270 0.07 · 40 = 2.8 L
g. 150% of 44 kg h. 200% of 56 students i. 2% of 1,500 km
1.50 · 44 kg = 66 kg 2 · 56 = 112 students 0.02 · 1,500 km = 30 km

Page 17
2.
a. 0.6 · 50 b. 0.03 · $400 c. 0.8 · 400 km
60% of 50 = 30 3% of $400 = $12 80% of 400 km = 320 km
d. 0.08 · 6 e. 0.11 · $300 f. 0.2 · 70 kg
8% of 6 = 0.48 11% of $300 = $33 20% of 70 kg = 14 kg

64 Percent (Blue Series)


Percentage of a Number: Using Decimals, cont.
Page 17
3. a. 0.17 · $4,500 = $765
b. 0.67 · 27 m = 18.09 m
c. 0.48 · 7.8 kg = 3.744 kg
4. a. 0.25 · 240 m = 60 m b. 0.80 · 30,000 km = 24,000 km c. 0.75 · 3.2 kg = 2.4 kg
5. a. Since the whole shoreline is 100%, the percentage of the shoreline that is not sandy beach is 100% − 6% = 94%.
b. 6% of 30 km means 0.06 · 30 km = 1.8 km of the shoreline is sandy beach.
6. a. 100% − 20% = 80% of the students do not have scholarships.
b. 0.20 · 4,000 = 800 students have scholarships.
c. 0.80 · 4,000 = 3,200 students do not have scholarships.
7. There were 360 hectares planted in wheat, 540 hectares in corn and 300 hectares in oats:
Crop Percentage Decimal Hectares Planted
Wheat 30% 0.30 360
Corn 45% 0.45 540
Oats 100% − 30% − 45% = 25% 0.25 300
TOTAL 100% 1.00 1,200

Page 18
8. a. Gladys did not convert 80% to a decimal. It should be 0.8 · 50 = 40.
b. Glenn did not continue his solution far enough. He only found 25% of 84,000. After finding 25% of 84,000, he should
have multiplied that by 3 to find 75% of 84,000. 84,000 ÷ 4 = 21,000, and 21,000 · 3 = 63,000.
9. Five of the expressions have the same value as 20% of $620:
0.02 · $620 $620 ÷ 5 $620 ÷ 10 · 2 2 · $62
1
5
· $620 0.2 · $620 20 · $620 $620 ÷ 4

10. About 19,200,000 more Italians than Tanzanians live in cities.


First calculate 69% of 60 million Italians: 0.69 · 60,000,000 = 41,400,000 Italians. Then calculate
37% of 60,000,000 = 22,200,000 Tanzanians. The difference is 41,400,000 − 22,200,000 = 19,200,000
more Italians than Tanzanians. You can also solve it by first subtracting 69% − 37% = 32% and
then multiplying 0.32 · 60,000,000.
11.

Activity Percentage Minutes Hours/minutes b.


Sleep 38% 547 9 h 7 min
School 21% 302 5 h 2 min
Soccer 10% 144 2 h 24 min
Play 11% 158 2 h 38 min
Eating 9% 130 2 h 10 min
Chores 9% 130 2 h 10 min
Hygiene 2% 29 29 min
TOTAL 100% 1,440 24 hours

65 Percent (Blue Series)


Discounts, pp. 19-20
Page 19
1. a. Sale price: $90 − $18 = $72.
b. Discount: 0.40 · $5 = $2. Sale price: $5 − $2 = $3.
c. Discount: 0.30 · $15 = $4.50. Sale price: $15 − $4.50 = $10.50.
2. Monica subtracted the percentage discount as a dollar amount instead of calculating the amount of the discount in
dollars. First she should have figured out 20% of $25, which is $5 (not $20). Then she should have subtracted
$25 − $5 = $20. So the correct discounted price is $20.
3.
a. Discount amount: $0.30 b. Discount amount: $4.50 c. Discount amount: $45
Discounted price: $0.90 Discounted price: $13.50 Discounted price: $105
d. Discount amount: $8 e. Discount amount: $0.40 f. Discount amount: $0.65
Discounted price: $12 Discounted price: $3.60 Discounted price: $0.65

Page 20
4. a. Round $39.90 ≈ $40. Then calculate 30% of $40 = $12. The estimated discounted price is $40 − $12 = $28.
b. Round 17% to 20%. Then calculate 20% of $12.50, which is $2.50. The estimated discounted price is $10.
Or round 17% to 20% and $12.50 to $13. Then 20% of $13 is 2.60, and the estimated discounted price is $10.40.
Or round 17% to 15% and $12.50 to $12. Then 15% of $12 is $1.80, and the estimated discounted price is $10.20.
Note: the exact answer is (1 − 0.17) · $12.50 = $10.38, so the second method, which calculates an estimated
discounted price of $10.40, is the most accurate. Why do you think that is? *
c. Round $75.50 to $80. Then 75% of $80 is $60, and the estimated discounted price is $20.
* Note to problem 4. b. (“Why do you think that is?”): By rounding the 17% discount to 20%, we are figuring ourselves
a bigger discount than what we would really get. So by rounding the price up to a higher price, from $12.50 to $13.00,
we are offsetting that error. When our errors of estimation cancel, our estimate is generally closer to the exact price.
5. a. Round $199 to $200. Since 75% of $200 is $150, the estimated discounted price would be $50, which still costs more
than the off-brand mp3 player for about $45. So you’ll need to decide if there is a big enough difference in quality
between the two brands to justify paying $5 more.
b. Round $89 to $90. Then 40% of $90 is $36, and the estimated discounted price is about $90 − $36 = $54, which is still
more than about $40 for a used copy. You will need to decide if the difference in condition is worth $14 to you!
6. They would earn more money by selling it at the 25% discount. Without the discount, they earn about 50 · $10 =
$500 a week. Discounting it to around $7.50 would make their estimated income about 100 · $7.50 = $750 a week.
7. a. The jeans were discounted by 10%. b. The phone was discounted by 25%. c. The haircut was discounted by 20%.
8. The expressions that are in grey will not work, but all of the others will.

$46 $46 $46 $46


0.25 · $46 0.75 · $46 $46 − 25
$46 − 4 4 4
·3

Practice with Percent, pp. 21-23


Page 21
1. a. 0.10 · $50 = $5 b. $10 / $50 = 1/5 = 20%
2. a. Jenny ate 0.60 · 25 = 15 cookies.
b. Jared ate 6/25 = 24/100 = 24% of the cookies.
3. a. Jack made baskets on 17/20 or 85% of his shots.
b. Jack made 0.56 · 50 = 28 baskets in all.
c. 0.60 · 25 = 15 of the women like chocolate.
d. 42/200 = 21/100 = 21% of the citizens voted for Mr. X.
e. 620/1,000 = 62/100 = 62% of the boxes contained books.
f. 0.14 · 50 = 7 of the participants came late.

66 Percent (Blue Series)


Practice with Percent, cont.
Page 22
4. a. 15/100 of the workers are over 50. (15 percent is
a portion of 100 percent, not of 40 workers.)
b. 6 workers are over 50. (15/100 · 40 workers
= 6 workers. The 15 is a percent, not a quantity
of workers.)
5. If Jose gave away 70% of his stuffed animals,
then he kept 30%, so he has 0.30 · 20 = 6 left.

6. Jerry painted 15 m ÷ 25 m = 3/5 = 0.6 = 60% of the fence.

7. If Marie wasted 30% of her money, then she has 70%


left. Since 0.70 · $20 = $14, she has $14 left.

8. a. 2/6 = 1/3 ≈ 33%.


b. 1/4 + 1/8 = 3/8 = 0.375 ≈ 38%.
c. 3/12 = 1/4 = 25%.
Page 23
9. a. Evelyn’s height is 1 m 4 cm = (100 + 4) cm = 104 cm. Mary’s height is 1 m 57 cm = (100 + 57) cm = 157 cm.
Thus Evelyn’s height is 104 cm / 157 cm ≈ 0.66 = 66% of Mary’s.
b. Jacqueline is 90% · 1 m 90 cm = 0.9 · 190 cm = 171 cm = 1 m 71 cm tall.
10. a. Student explanations will vary; check the student’s explanation. For example:
Peter has to use a larger share of his wages because the two boys pay the same, but he makes less.
Or, 225/900 is a bigger fraction than 225/1,350 because the numerators are the same but the denominator is smaller.
b. Each boy pays 50% of $450, or $225, a month for rent. So Peter pays $225/$900 = 25% of his wages,
and Jake pays $225/$1,350 ≈ 17% of his wages.
11. a. 11% of $402 is more. You can estimate that 11% of $402 is about 10% of $400, which is $40.
Similarly, 12% of $298 is about 10% of $300, which is $30.
b. Since 10% of $400 is $40 and 10% of $300 is $30, the difference is about $10.
12. a. The average distance that they traveled was the length of the total trip divided by how long it took them to travel it,
or 1 920 kilometers / 4 days = 480 kilometers per day.
b. Day 1 544 kilometers / 1 920 kilometers = 0.2833... ≈ 28%
Day 2 448 kilometers / 1 920 kilometers = 0.2333... ≈ 23%
Day 3 640 kilometers / 1 920 kilometers = 0.3333... ≈ 33%
Day 4 288 kilometers / 1 920 kilometers = 0.15 = 15%
c. If they had divided the trip into four equal portions, on each day they would have driven 1/4 = 25% of the total trip.

Finding the Total When the Percentage Is Known, pp. 24-25


Page 24
1. Margie had 200 marbles. If 20% is 40 marbles, then 10% is 20 marbles. And 100% will be 10 times that,
or 200 marbles. In a bar model, draw 5 blocks, one block representing 40 marbles.
2. One-half cup is 25% of the original amount. This means the original amount was four times that much, or two cups.
3. Eric had $750. If 12% is $90, then 1% is $90 ÷ 12 = 7.5. And 100% is 100 times that, or $750.

67 Percent (Blue Series)


Finding the Total When the Percentage Is Known, cont.
Page 25
4. The dress cost $30 before the discount. The discounted price of $24 is 80% of the price. So, 10% of the price is
$24 ÷ 8 = $3, and 100% of the price is $30.
5. The ticket price was $54 before the discount. The discounted price of $21.60 is 40% of the original price.
So, 10% of the price is $21.60 ÷ 4 = $5.40. Therefore, 100% of the price is 10 times that, or $54.
6. Joe had $200. What he has left ($56) is 28% of his original money. So, 1% of his money is $56 ÷ 28 = $2 , and
100% of his money is 100 times that, or $200.
7. She spent $130. What she has left ($120) is 48% of her original money. So, 1% of her original money is $120 ÷ 48
= $2.50. Originally she had 100 times that, or $250. So, she spent $250 − $120 = $130.
8. He has 1,080 Herefords. To get 1% of his cows, we divide 420 ÷ 28 = 15. This means he has 1,500 cows in total
(since 100% of his cows is 100 · 15 = 1,500.) And thus, he has 1,500 − 420 = 1,080 Herefords.
9. There were 150 people who had bought that brand of coffee grinder. The 126 people who were happy make up 84%
of the total. This means 1% is 126 ÷ 84 = 1.5, and 100% is a hundred times that, or 150 people.
Puzzle corner. Because both calculators have the same discounted price of $42, the calculator with the smaller discount
(25%) was originally cheaper. Since $42 is 70% of the price of the one calculator, then $42 ÷ 7 = $6 is 10% of the original
price. And thus the original price was 10 · $6 = $60. Then, since $42 is 75% of the price of the other calculator, divide $42
÷ 3 = $14 to find 25% of the original price. Next, multiply that by 4 to find the original price of $56. Thus the calculator that
was discounted by 25% was originally $60 − $56 = $4 cheaper.

Review: Percent, pp. 26-27


Page 26
1.
68 7 15
a. 68% = = 0.68 b. 7% = = 0.07 c. 15% = = 0.15
100 100 100
120 224 6
d. 120% = = 1.20 e. 224% = = 2.24 f. 6% = = 0.06
100 100 100

2.
percentage / number 6 100 90 57 6
1% of the number 61 0.9 0.57 0.06

4% of the number 244 3.6 2.28 0.24

10% of the number 610 9 5.7 0.6

30% of the number 1,830 27 17.1 1.8

3. There are 15 + 5 = 20 skaters. So 15/20 = 75% of the skaters are girls.


Page 27
4. a. 75% b. 8% c. 163%
5. Emma’s height is 134% of Madison’s height. 163/122 ≈ 134.
6. The other chair costs $35. 1.4 · 25 = 35.
7. 25 = 1/5 of the marbles. 5 · 25 = 125 marbles total. 4 · 25 = 100 white marbles.
8. 540/2,000 = 54/200 = 27/100 = 0.27. Andrew pays 27% of his salary in taxes.
9. Since 0.80 · $18 = $14.40 and 0.90 · $16 = $14.40, they are both the same price.
10. The area of the square with 2-cm sides is 4 cm2, and the area of the square with 4-cm sides is 16 cm2.
Because 4/16 = 0.25, the area of the smaller square is 25% of the area of the larger square.

68 Percent (Blue Series)


Percentage Basics, pp. 28-30
Page 28
1.
52 7 59
a. 52% = = 0.52 b. 7% = = 0.07 c. 59% = = 0.59
100 100 100
109 382 200
d. 109% = = 1.09 e. 382% = = 3.82 f. 200% = = 2.0
100 100 100

2.
282 67 891
a. 28.2% = = 0.282 b. 6.7% = = 0.067 c. 89.1% = = 0.891
1000 1000 1000
9 1039 3409
d. 0.9% = = 0.009 e. 10.39% = = 0.1039 f. 340.9% = = 3.409
1000 10000 1000
g. 45.39% = 0.4539 h. 2.391% = 0.02391 i. 94.2834% = 0.942834

Page 29
3.
8 32 142 71 24 120
a. = = 32% b. = = 71% c. = = 120%
25 100 200 100 20 100

4.

a. 11/8 = 137.5% 1.3 7 5 b. 11/24 ≈ 45.8% 0.4 5 8 3


8 ) 1 1.0 0 0 24 ) 1 1. 0 0 0 0
- 8 - 96
30 140
-2 4 -1 2 0
60 200
-5 6 - 192
40 80
-40 -72
0 8

5.
2 6
a. ≈ 0.6667 = 66.67% b. ≈ 0.8571 = 85.71%
3 7
17 52
c. ≈ 0.7391 = 73.91% d. ≈ 0.5306 = 53.06%
23 98

Page 30
6.
Ocean Area / Total Area Percent
Pacific 155,557,000 km2 / 335,258,000 km2 = 0.46399... 46.4%
Atlantic 2
76,762,000 km / 335,258,000 km = 0.22896... 2 22.9%
Indian 68,556,000 km2 / 335,258,000 km2 = 0.20448... 20.4%
Southern 20,327,000 km2 / 335,258,000 km2 = 0.06063... 6.1%
Arctic 14,056,000 km2 / 335,258,000 km2 = 0.04192... 4.2%

69 Percent (Blue Series)


Percentage Basics, cont.
Page 30
7. The area of the Carters’ land is 85.5% of that of the Joneses’.
The area of the Carters’ land is 40 m · 35 m = 1,400 m2. The area of the Joneses’ land is 42 m · 39 m = 1,638 m2.
So the area of the Carters’ land is 1,400/1,638 ≈ 0.8547 ≈ 85.5% of the area of the Jones’s land.
8. a. The weight of Captain is 140% of the weight of Chief.
b. Captain weighs 140% · 6 lb = 1.4 · 6 lb = 8.4 lb.
Puzzle corner.
a. There are two quarters fully uncolored and there is one quarter fully colored. The fourth quarter is
half colored. (Count the little squares: 8∕16 = ½). So one full quarter (two-eighths) and half of another
quarter (one more eighth) are colored. As a percentage, 3/8 = 0.375 = 37.5%.

b. The blue rectangle on the left is half of the square. The other rectangle on the right is also half of
the square. In that rectangle on the right, the four small triangles have the same area as the two larger
ones. (Imagine that the entire square is divided into quarters.) So the three blue triangles make up ¾
of half of the area of the right rectangle, so they make up ¾ of ½ of ½ of the entire square. As a
percent, the blue-shaded area is the sum of the triangles and the rectangle:
(¾ · ½ · ½) + (½) = 3∕16 + 8∕16 = 11∕16 = 0.6875 = 68.75%.

Solving Basic Percentage Problems, pp. 31-33


Page 31
1.
a. 107% = 1.07 b. 16.67% = 0.1667 c. 4.5% = 0.045

2. a. Laptop, original price $249.90, increase 6%.


New price = 1.06 · $249.90 = $264.90.
b. Biology textbook, original price $82.40, increase 2.5%.
New price = 1.025 · $82.40 = $84.46.
c. Lunch buffet, original price $18.50, increase 11.2%.
New price = 1.112 · $18.50 = $20.57.
3. Part / Total = $325.08 / $1,890 = 0.172 = 17.2%. Julia paid 17.2%.
Page 32
4. The new price is $39.59.

Since 13% of the price is removed, 87% of the price is left. I write that percentage as
a decimal and multiply the original price by it: 0.87 · $45.50 = $39.59.

5. a. New price = 0.62 · $78 = $48.36


b. New price = 0.775 · $14.95 = $11.59
6. a. 9 in : 18 in = 1:2.
b. Since the sides are in a ratio of 1:2, the unknown side of the larger rectangle is 30 in.
So the area of the smaller rectangle is 15 in · 9 in = 135 sq in and of the larger is 30 in · 18 in = 540 sq in.
Thus the two areas are in a ratio 135 : 540 = 1:4. (Notice that the ratio of the areas is just the square of the ratio of
the sides: ½ · ½ = ¼.)

c. The area of the smaller rectangle is 135 cm2 / 540 cm2 = 25% of the area of the larger rectangle.

70 Percent (Blue Series)


Solving Basic Percentage Problems, cont.
Page 32
7.
a. Bicycle: $100; 7% sales tax. b. Fridge: $400; 6% sales tax. c. Haircut: $50; 3% sales tax.
Tax to add: $7 Tax to add: $24 Tax to add: $1.50
Price after tax: $107 Price after tax: $424 Price after tax: $51.50

Page 33
8.
a. A microscope. b. 2 shirts. c. Crayons and a toy horse.

$180 + $9 = $189 2($18 + $0.90) = $37.80 $2 + $13 = $15


$15 + $0.75 = $15.75
d. A bag and a shirt. e. 6 bottles of water. f. 2 shirts and a bag.

$70 + $18 = $88 6 · $2 = $12 $18 + $18 + $70 = $106


$88 + $4.40 = $92.40 $12 + $0.60 = $12.60 $106 + $5.30 = $111.30

9. The final price of a music CD is $11.50 + $0.77 = $12.27.


10. Jeremy will pay $850 · (1 − 0.375) = $531.25 for the vacation package.
11. a. The total price Patrick paid was 1.012 · $200,000 = $202,400.
b. 2,000 m2 / 5,000 m2 = 2/5 = 40%; 0.4 · $202,400 = $80,960.
Patrick should charge his neighbor $80,960 in order to recover what
he paid for the parcel that he is selling him.
12. a. The final price of the CD is 0.8 · $12.50 + 0.07 · $10 = $10.70.
b. The final price of the jeans is 0.6 · $55.97 + 0.05 · $33.58 = $35.26.
13. a. The average price of the four types of pasta is $1.85.
b. The average price would be $1.67 after the discount.

Percent Equations, pp. 34-38


Page 34
1. a. headphones, price $12, discounted by 24%. New price = 0.76 · $12
b. a bag of dog food, price p, discounted by 11%. New price = 0.89p
c. pizza sauce, price x, discounted by 17%. New price = 0.83x
d. sunglasses, price s, price increased by 6%. New price = 1.06s

2. p − 0.25p = $576
0.75p = $576 ÷ 0.75
p = $768
The computer cost $768 before the discount.

Page 35

3. r + 0.05r = $215.25
1.05r = $215.25 ÷ 1.05
1.05r / 1.05 = $215.25 / 1.05
r = $205.00
The rent was $205 before they raised it.

71 Percent (Blue Series)


Percent Equations, cont.
Page 35
4. a. 2(p − 0.3p) = 98

b. 2(p − 0.3p) = $98 ÷2


2(p − 0.3p) / 2 = $98/2
p − 0.3p = $49
0.7p = $49 ÷ 0.7
p = $49/0.7 = $70

The original price was $70 per tablet.


5. a. (1 + 0.065 + 0.0085)p which simplifies to 1.0735p .

b. 1.0735p = $16,639.25 ÷ 1.0735


p = $16,639.25/ 1.0735
p = $15,500

Page 36
6. a. The percent proportion is part/total = percent/100. In this case, we have (discounted price)/(original price) = 78/100,
so the correct equation is 28/p = 78/100.
$28 78
b. =
p 100

78p = 100 · $28

78p = $2,800

78p $2,800
=
78 78

p = $35.90
Page 37
d 56
7. a. =
4,500 km 100

100d = 56 · 4,500 km

100d = 252,000 km

100d 252,000 km
=
100 100

d = 2,520 km

72 Percent (Blue Series)


Percent Equations, cont.
Page 37
7. b. Because here the unknown is the whole (100%), the value 6.08 is the part (38%). Thus the 6.08 and the 38%
both go on the top of the proportion, and the unknown number n and the 100% go on the bottom, like this:
6.08 38
=
n 100

38n = 100 · 6.08

38n = 608

38n 608
=
38 38

n = 16
8. If the conditioner was 15% off, it cost 100% − 15% = 85% of its regular price. Also, with the discount a single
bottle cost $50.79 / 5 = $10.158. So the proportion is (discounted price)/(regular price) = 85/100. Let b be the
regular price of one bottle. Then the proportion becomes:
$10.158 85
=
b 100

85b = 100 · $10.158

85b = $1015.80

85b $1015.80
=
85 85

n ≈ $11.95
The price of one bottle of hair conditioner before the discount was $11.95. You can of course solve this problem in
other ways, as well.
Page 38
9. The part is the tax, and the total is the value, v. The percent proportion is $95.20/v = 0.8/100. You can also solve this
by writing the equation 0.008v = $95.20. Either way, the solution is v = $11,900.
The tax agency valued his land at $11,900 .
10. If the price of electricity was lowered by 5%, it is now 95% of what it was before, so we can write a percent proportion
now/then = $0.133 / p = 95/100. Or, you can write the equation 0.95p = $0.133. Either way, the solution is p = $0.140.
The price of electricity was $0.14 per kilowatt-hour before the decrease in cost.
11. The price after the discount ends up being $4.55.
The price after the 30% increase is 130% of the original price: $5 · 1.3 = $6.50. The price after the 30% discount is
70% of that higher price: $6.50 · 0.7 = $4.55. The prices are different because the 30% discount affects a bigger price
than the 30% increase did (it is a discount not only on the original price but also on the increase), so the discount ends
up being bigger than the increase. In this case, the customer got only a $0.45/$5.00 = 9% discount from the original
$5.00 price.

73 Percent (Blue Series)


Percent Equations, cont.
Page 38
12. Answers will vary. Please check the student’s work.
Substituting numbers: Suppose the base b were 3 km and the height h were 4 km to give A = ½ bh
= ½ (3 km · 4 km) = 6 km2. Increasing 3 km and 4 km by 10% gives the increased area A′ = ½ (3.3 km · 4.4 km)
= ½ (14.52 km2) = 7.26 km2, which is an increase of 1.26 km / 6 km = 21%.
Using algebra: The formula for the area of a triangle is A = ½ bh. Let's use b and h for the base and height of
the original triangle, and b' for the base and h' for the height of the bigger triangle.

Because the increase is 10%, we know that b' = 1.1b and h' = 1.1h. If we substitute those into the formula for the area,
we get the area of the bigger triangle as Abig = ½ b'h' = ½ (1.1b)(1.1h) = 1.21 (½ bh). In the last expression, the part
½ bh is the area of the original triangle, so we can write Abig = 1.21 (½ bh) = 1.21 A, where A is the area of the
original triangle.

So, since area of the bigger triangle is 1.21A, that means it is 121% of A, which means the increase in area is 21%.
Puzzle Corner: Let’s use m to represent the monthly water bill before the increase. The bill after the 10% increase is 1.1m.
For 7 months, January through July, the bill is m, for a total of 7m. For 5 months, August through December, the bill is
1.1m, for a total of 5 · 1.1m = 5.5m. The total of the bills for the whole year is thus 7m + 5.5m = 12.5m = $584.00. Therefore
the monthly water bill before the increase was: m = $584.00 / 12.5 = $46.72.

Circle Graphs, pp. 39-40


Page 39

1. a. b. c.

2. Type Quantity Fraction Percent Central Angle


white bread 50 1/4 25% 90º
bran bread 25 1/8 12.5% 45º
rye bread 30 3/20 15% 54º
corn bread 40 1/5 20% 72º
4-grain bread 55 11/40 27.5% 99º
TOTAL 200 1 100% 360º

3. a.

b. No, it does not.

74 Percent (Blue Series)


Circle Graphs, cont.
Page 40
4. Answers will vary. The answers below are actually the exact percentages. However, students’ answers may differ from
these a little and be totally acceptable. (Each circle should sum to 100%.) Starting from 12:00 and going clockwise:
a. About 45%, about 5%; about 40%, about 10%. (The first two and the last two should each sum to 50%.)
b. About 20%, 20%, 10%; 25%, 12.5%, 12.5%. (The first three and the last three should each sum to 50%.)
c. About 65%, about 20%, about 15%.

5. Amount Percent Central


Flavor
sold of total Angle
chocolate 67 46.9% 169º
vanilla 34 23.8% 86º
strawberry 16 11.2% 40º
blueberry 26 18.2% 65º
TOTAL 143 100% 360º
Note: The “Percent of total” column actually totals to 100.1% because the numbers that were rounded up moved a
little bit farther in the upward direction than the numbers that were rounded down moved in the downward direction.
That is typical for calculations that round several numbers.

6. Favorite hobby Percent Central Angle


Reading 12.3% 44º
TV 24.5% 88º
Computer games 21% 76º
Sports 22.3% 80º
Pets 7.1% 26º
Collecting 8.1% 29º
no hobby 4.7% 17º
TOTAL 100% 360º

Percentage of Change, pp. 41-43


Page 41
1. a. Discount: difference/original = ($12 − $8) / $12 = $4 / $12 = 0.3 = 33.3%.
b. Discount: difference/original = ($20 − $16) / $20 = $4 / $20 = 0.2 = 20%.
c. Increase: difference/original = ($20 − $15) / $15 = $5 / $15 = 0.3 = 33.3%.
d. Increase: difference/original = ($160 − $200) / $160 = $40 / $160 = 0.25 = 25%.
Page 42
2. a. Since 25% of $24 is $6, the discounted price is $24 − $6 = $18.
b. The difference is $24 − $18 = $6, so the percentage of discount is $6/$24 = 0.25 = 25%.
3. a. The difference is 6 kg − 5 kg = 1 kg, so the percentage of increase is 1 kg/5 kg = 0.20 = 20%.
b. Since 20% of 6 kg is 1.2 kg, his weight at six months is 6 kg + 1.2 kg = 7.2 kg.
4. The difference from June to July is $342 − $325 = $17, and the percentage change is measured
relative to the rent for the month of June. So the percentage of increase is $17/$325 ≈ 0.052 = 5.2%.
The difference from July to August is $349 − $342 = $7, and the percentage change is measured
relative to the rent for July. So the percentage of increase is $7/$342 ≈ 0.020 = 2.0%. Therefore,
the change from June to July is a greater percentage increase than the change from July to August.

75 Percent (Blue Series)


Percentage of Change, cont.
Page 43
5. a. The change is $60 − $54 = $6, so the percentage decrease is $6/$60 = 0.10 = 10%.
b. The change is still $60 − $54 = $6, but this time the percentage of increase is measured from the
lower price. So the percentage of increase is $6/$54 = 0.1 ≈ 11.1%. The reason the percentage
changes are not the same is that we are measuring them relative to different prices.
6. a. Calculate the final price: Since 20% of $50 is $10, after the increase the price was $50 + $10 = $60.
The following decrease in price is measured relative to that $60. Since 20% of $60 is $12, the final
price is $60 − $12 = $48.
b. Calculate the true percentage change: Since $50 − $48 = $2, the true percentage change is a decrease
of $2/$50 = 0.04 = 4%. The final price is lower because the 20% discount ended up being larger than
the 20% increase. It was larger because it was calculated relative to a higher price.
7. Jake’s hours were cut by (40 − 37.5)/40 = 0.0625 = 6.25%. Anita’s were cut by (145 − 135)/145 ≈ 0.069 = 6.9%.
So Anita’s hours were cut by a larger percentage.
8. a. The first increase (by 20%) raised the price to 1.2 · $100 = $120. The second increase (by 10%) started from
$120, so it raised the price to 1.1 · $120 = $132.
b. If the price had gone from $100 to $132 in one step, the increase would have been $32/$100 = 0.32 = 32%.

Percentage of Change: Applications, pp. 44-47


Page 44
1. The area increases by 125%.
The old area is 10 m · 10 m = 100 m2. The new area is 15 m · 15 m = 225 m2. The difference is 225 − 100 = 125 m2.
To find the percent increase, we calculate the fraction 125/100 = 125%.
2. The area will decrease by 29.5%.
The original area is 21 cm · 29.7 cm = 623.7 cm2. The new area is 17.6 cm · 25 cm = 440 cm2. The difference is
623.7 − 440 = 183.7 cm2. The percentage of decrease is the fraction 183.7/623.7 = 0.2945... ≈ 29.5%.
3. a. If the length of the side before scaling is x, then the length of the side after scaling is 1.15x.
So the side increases by 1.15x − 1.00x = 0.15x. So as a percentage the increase is 0.15/1.00 = 15%.
b. Let’s say the side of the square measures 100 cm. After the increase, the side measures 115 cm. The area at first is
100 cm · 100 cm = 10,000 cm2. After the increase, the area is 115 cm · 115 cm = 13,225 cm2. The percentage
increase in area is 3,225/10,000 = 0.3225 = 32.25%. Rounded to the nearest tenth of a percent, that’s 32.3%.
You can also calculate the percentage of increase by reasoning using variables and some algebra. Let s be the
side of the square before the increase. After the increase, the side is 1.15s. The area at first is s2, and after the
increase it is 1.152s2 or 1.3225s2. The percentage increase in area is (difference in area)/(original area)
= (1.3225s2 − s2)/s2 = 0.3225s2/s2 = 0.3225 = 32.25%. Again, that’s 32.3% to the nearest tenth of a percent.
c. No, the answer won’t change. The percentage increase in area is still 32.35% whether we have a rectangle or a
triangle. Whether we are calculating the area of a square, rectangle, or triangle, we multiply two dimensions:
either the base and the altitude or the two sides. In all cases, the two dimensions end up being multiplied by 1.15,
which means the area increases by the factor 1.15 · 1.15 = 1.3225. That scale factor corresponds to an increase
of 32.25%.
Page 45
4. a. Answers will vary. Please check the student’s answer.
For example: More water is used in July because it is hotter.
b. The decreases are: For June, (215,363 − 205,849) / 215,363 = 4.0%. For July, (237,558 − 219,875) / 237,558 = 7.4%.
For August, (231,777 − 204,831) / 231,777 = 11.6%.
c. The greatest percentage decrease in water production was in August.
On the graph you can see that the bar for August 2014 is less than half the length of the bar for August 2013, whereas
for June and July, the bars for 2014 are more than half the corresponding bars for 2013.

76 Percent (Blue Series)


Percentage of Change: Applications, cont.
Page 45
5. The last increase would be less than the first. When the price went from $5.50 to $6.00, the percent increase was
$0.50/$5.50 ≈ 9.1%. When it went from $6.00 to $6.50, the increase was $0.50/$6.00 ≈ 8.3%. If it goes from $6.50
to $7.00, the increase will be $0.50/$6.50 ≈ 7.7%, which is less than the first increase. Even though the actual amount
of increase ($0.50) stays the same, the base price ($7.00) that we are comparing to is now larger than it was ($5.50)
in the first increase. The value of the fraction 0.5/7 is smaller than the value of the fraction 0.5/5.5.
Page 46
6. a. The price before tax is $69.99 / 1.07 ≈ $65.41.
b. The percentage of increase in price with tax is ($79.99 − $69.99)/$69.99 ≈ 14.3%.
c. The percentage increase in price without tax is also about 14.3%.
The new price before tax is $79.99/1.07 = $74.76. The percentage of increase is ($74.76 − $65.41)/$65.41 ≈ 14.3%.

7. a. The area increases by 24.9%. The original windows have the area 85 cm · 85 cm = 7,225 cm2.
The new windows will have the area 95 cm · 95 cm = 9,025 cm2. The difference is 9,025 − 7,225 = 1,800 cm2.
To find the percentage of increase, calculate the fraction 1,800/7225 = 0.2491... ≈ 24.9%.
b. Approximately 16 of the larger windows cover the same area as 20 of the smaller ones. Twenty of the smaller
windows cover an area of 7,225 cm2 × 20 = 144,500 cm2. Dividing by the area of the larger window gives the
number of larger windows needed to cover that area: 144,500 cm2 / 9,025 cm2 = 16.01 ≈ 16.
8. a. The $50 item has the greatest percent increase in price. When we compare the fractions 10/50, 10/60, and 10/70,
the fraction 10/50 is the biggest because it has the smallest denominator.
b. The price of the $70 item decreases the most in dollars. When we calculate the dollar increases, 0.12 · $50,
0.12 · $60, and 0.12 · $70, the last one is the biggest.
Page 47
9. During that decade the population of the state of Kentucky increased by 3,219,000 − 3,038,000 = 181,000 people.
As a percentage of the 1960 population, that increase is 181,000 / 3,038,000 = 0.05957... ≈ 6.0%.
10. Increase for Ratio % increase
Year Population
the decade Increase / Population for the decade
1960 3,038,000 — — —
1970 3,219,000 181,000 0.05957... 6.0%
1980 3,661,000 442,000 0.13730... 13.7%
1990 3,685,000 24,000 0.00655... 0.7%
2000 4,042,000 357,000 0.09687... 9.7%
2010 4,340,000 298,000 0.07372... 7.4%

11. a. Which decade? From 1970 to 1980.


b. The slope of the line is the steepest during that decade.
12. Answers will vary. Please check the student’s work.

Comparing Values Using Percentages, pp. 48-51


Page 48
1. a. difference/reference = 2 m / 6 m = 1/3 = 33.3%
b. 180 cm / 120 cm = 3/2 = 150%
c. 1 m / 4 m = 25%

77 Percent (Blue Series)


Comparing Values Using Percentages, cont.
Page 49
2. Heather is 20/140 = 14.3% taller than Erica. Erica is 20/160 = 12.5% shorter than Heather.
3. a. (1) The population of Tokyo is (37.80 − 21.75)/ 21.75 = 0.7379… ≈ 73.8% larger than the population of Delhi.
(2) The population of Moscow is (21.60 − 15.51)/21.60 = 0.2819… ≈ 28.2% smaller than the population of
Mexico City.
b. The difference between the populations of Seoul and New York is (25.62 − 19.95)/19.95 = 0.2842… ≈ 28.4%.
The difference between the populations of New York and Moscow is (19.95 − 15.51)/ 15.51 = 0.2863… ≈ 28.6 %.
So the difference between the populations of New York and Moscow is larger by 0.2% (which is probably much less
than the error in measuring the original numbers).
14,000 − 10,000 4,000
4. The difference in area is = = 33%.
12,000 12,000
$32 − $28 $4
5. a. The percentage of difference in cost for labor is = ≈ 13.3%.
$30 $30
$64 − $56 $8
b. The percentage of difference in cost for labor is the same: = ≈ 13.3%.
$60 $60

Page 50
6. a. The shorter plant’s height is 12 cm / 16 cm = 3/4 = 75% of the taller plant’s height.
b. Write the fraction (difference)/(reference), using the shorter plant’s height as the reference.
The taller plant is 4 cm / 12 cm = 1/3 ≈ 33.3% taller than the shorter plant.
c. Again, write the fraction (difference)/(reference), but using the taller plant’s height as the reference.
The shorter plant is 4 cm / 16 cm = 1/4 = 25% shorter than the taller plant.
d. Again, write the fraction (difference)/(reference), but using the average height (14 cm) of the two plants as the
reference. The average height is: ½(12 cm + 16 cm) = 14 cm. So the relative difference between their heights is
4 cm / 14 cm = 2/7 ≈ 28.6% .
Page 51
7. a. The relative height of Baby’s tower is: (height of Baby’s tower)/(height of Jack’s tower) = 30 cm / 150 cm
= 10/50 = 20/100 = 20% of the height of Jack’s tower, so Mary is right. Elijah subtracted two heights to get a
percentage; the difference in height is 120 cm, but that wasn’t what the question asked for. Angela compared the
difference in height instead of the height of the tower that Baby built.
b. The school orchestra has: (difference)/(number of girls) = (26 − 14)/(14) = 12/14 = 0.85714... ≈ 85.7% ≈ 86%
more boys than girls, so Elijah is right. Angela subtracted two counts to get a percentage; there are 12 boys more,
not 12% more. Mary wrote correctly what percentage the number of girls is of the number of boys, but
that wasn’t what the question asked for.
8. a. See the table on the right.
Number of violent crimes in 2013-2014
b. You might move into County Z because of the slower increase County A County Z
of the crime rate, but 512 crimes is so many more than 4 that
2013 2 454
County A might well be a safer area to move into. However, it
would also be good to know the total population of the areas. If 2014 4 512
there were only 4 crimes but a population of 10 in County A and Percentage
512 crimes but a population of 30 million in County Z, you might 100% 12.8%
of increase
still be safer in County Z.
Puzzle corner:
In Ceredigion, the decrease was about 40 cases out of 93... which is nearly 1/2 or 50%.
In Conwy, the decrease is less than 30 out of 256, or barely over 10%.
In Gwynedd, the decrease is 62/200, which is 31%.
Clearly the percent of decrease was greatest in Ceredigion.

78 Percent (Blue Series)


Simple Interest, pp. 52-57
Page 52
1. a. Interest: I = P · r · t = $5,000 · 0.03 · 1 = $150. Total to withdraw: $5,150.
b. Interest: I = P · r · t = $3,500 · 0.043 · 4 = $602. Total to withdraw: $4,102.
c. Interest: I = P · r · t = $20,000 · 0.076 · 10 = $15,200. Total to withdraw: $35,200.
Page 53
2. Savings account: The interest earned is I = P · r · t = $3,000 · 0.034 · 3 = $306. CD:
Since the penalty is 6 months interest, in three years Sandy would earn 2.5 year’s interest.
The total interest earned is therefore I = P · r · t = $3,000 · 0.0392 · 2.5 = $294.
Sandy is better off using the savings account.
3. a. The 12.9% interest rate works out to 12.9%/12 = 1.075% a month. For a $450 tablet, with r per
month and t in months (the time units have to cancel to leave only dollars), for one month that is
I = P · r · t = $450 · 0.01075 · 1 = $4.8375.
b. A day is 1/365 of a year, so each day she pays I = P · r · t = $450 · 0.129 · (1/365) ≈ $0.1590 = 15.9¢.
Page 54
4. Since the rate is per month, the time needs to be in months, too. So you will
pay I = P · r · t = $690 · 0.0109 · 24 = $180.504 in interest.
5. Interest for the 10-month loan: I = P · r · t = $850 · 0.108 · 10/12 = $76.50.
Interest for the 7-month loan: I = P · r · t = $850 · 0.095 · 7/12 = $47.10.
For the shorter-term loan he would have paid $76.50 − $47.10 = $29.40 less interest.
6. Interest during the first year: I = P · r · t = $26,000 · 0.0275 · 1 = $715
Interest during the last 1.5 years: I = P · r · t = $26,000 · 0.0995 · 1.5 = $3,880.50.
The total to pay back is $715 + $3,880.5 + $26,000 = $30,595.50.
Page 55
7. a. The time t is 1 year, the principal is $1,000, and the interest is $45. From the formula I = prt, we get
$45 = $1,000 · r · 1 or $45 = $1,000r, from which r = 45/1000 = 0.045 = 4.5%.
b. The interest was $3,600, the time is 5 years, and the principal is $12,000. Using the formula I = prt, we get
$3,600 = $12000 · r · 5 or $3,600 = $60,000r. From that, r = 3,600/60,000 = 36/600 = 6/100 = 6%.
Page 56
8. Let p be the original principal. In 10 years, and at 6% interest rate, that principal earns an interest of I = prt
= p · 0.06 · 10 = 0.6p. In 10 years her account had $12,000, which is the original principal plus interest, or p + 0.6p.
We get the equation
p + 0.6p = $12,000
1.6p = $12,000
p = $12,000/1.6 = $7,500
The original principal was $7,500.

9. Using the formula I = prt we get the equation $500 = $2,000 · 0.115 · t. Here is the solution:
$500 = $2,000 · 0.115 · t
$500 = $230t
$230t = $500
t = $500/$230 = 50/23 ≈ 2.174 years

You would have to invest it for 2.174 years or about 2 years 2 months.

79 Percent (Blue Series)


Simple Interest, cont.
Page 56

10. Using the formula I = prt we get the equation $350 = $1,800 · r · 2. Here is the solution:
$350 = $3,600r
$3,600r = $350
r = $350/$3,600 = 35/360 ≈ 0.0972
You would need an interest rate of 9.72%.

Page 57
11. The interest they paid was $4,741.48 − $4,055 = $686.48. Using the formula I = prt, we get:
$686.48 = $4,055 · 0.1195 · t
$686.48 = $484.5725t
$484.5725t = $686.48
t = $686.48/$484.5725 ≈ 1.41667 years = 1 year 5 months
You can also solve the problem this way. The interest paid was $686.48. The interest rate of 11.95% per year is very close
to 1% per month, which would mean paying $40.55 per month. It would take approximately $686.48 /$40.55 ≈ 17 months
to pay off the vacation package.
Puzzle corner. a. Let’s look at just one example of how compound interest is calculated. Jayden buys a $5,000 motorcycle
on a credit card that has a 6% annual interest rate. Compounding the interest means that the interest is added to the principal
at certain intervals, in this case each month.

Month Monthly interest Principal


0 $5,000
1 $5,000 · 0.005 = $25 $5,025
2 $5,025 · 0.005 = $25.125 $5,050.125
3 $5,050.125 · 0.005 ≈ $25.251 $5,075.376
4 $5,075.376 · 0.005 ≈ $25.377 $5,100.753
5 $5100.753 · 0.005 ≈ $25.504 $5,126.2567
6 $5126.257 · 0.005 ≈ $25.631 $5,151.888
7 $5151.888 · 0.005 ≈ $25.759 $5,177.647
8 $5177.647 · 0.005 ≈ $25.888 $5,203.535

b. At the end of 2 years, Jayden would pay back $5,000 · 1.00524 = $5,635.80.

Final Review, pp. 58-59


Page 58
1. a. 0.092 · $150 = $13.80
b. 0.458 · 16 m = 7.328 m
c. 0.006 · 700 mi = 4.2 mi.
2. a. $9 · 0.80 = $7.20. New price: $7.20. b. $6 · 0.75 = $4.50. New price: $4.50. c. $90 · 0.70 = $63. New price: $63.
3. 0.82p = $23.37
0.82p/0.82 = $23.37/0.82
p = $28.50

80 Percent (Blue Series)


Final Review, cont.
Page 58
4. The price of the computer with tax is $459 · 1.07 = $491.13.
Andy’s share of it is $491.13 · 0.4 = $196.45.
Jack’s share of it is $491.13 · 0.6 = $294.68.
5. The discount percentage is $27/$180 = 3/20 = 0.15 = 15%.
Page 59
6. a. The scale factor between the triangles is 21 cm / 7 cm = 1/3. The unknown side is therefore 12 cm/3 = 4 cm.
The area of the larger triangle is 12 cm · 21 cm/2 = 126 cm2.
The area of the smaller triangle is 4 cm · 7 cm/2 = 14 cm2.
The area of the smaller triangle is 14/126 = 11.1% of the area of the larger triangle.
b. If you take the ratio from the larger to the smaller triangle, the sides of the triangles are in the ratio of 3:1.
Going from the smaller one to the larger one, the ratio is 1:3.
c. If you take the ratio from the larger to the smaller triangle, the areas are in a ratio of 126:14 = 63:7 = 9:1.
Going from the smaller one to the larger one, the ratio is 1:9.

7. Originally, the area of the wall painting would have been 5 m · 3 m = 15 m2.
After scaling, the sides are 5 m · 1.2 = 6 m and 3 m · 1.2 = 3.6 m, so the enlarged area is 6 m · 3.6 m = 21.6 m2.
The difference in area is 21.6 m2 − 15 m2 = 6.6 m2, so the percentage increase is (difference in area)/(original area)
= 6.6 m2/15 m2 = 6.6/15 = 0.44 = 44%. An easier way to figure this is just to realize that, regardless of the actual
dimensions, since the scaling of each side is 1.2, the scaling of the area is just 1.2 · 1.2 = 1.44, so the increase in
area is 44%.
8. The difference in their times is 200 sec − 120 sec = 80 sec. Their average time was ½(200 sec + 120 sec) = 160 sec.
a. (Difference)/(The Old Gray Mare’s time) = 80/200 = 4/10 = 40%. Old Paint was 40% quicker than the Old
Gray Mare.
b. (Difference)/(Old Paint’s time) = 80/120 = 2/3 = 66.7%. The Old Gray Mare was 66.7% slower than Old Paint.
c. (Difference)/(Average time) = 80/160 = 1/2 = 50%. The relative difference between the two horses was 50%.
9. Assuming none of the interest was added to the principal during the time of the loan, he would owe the total amount
of interest at the end of two years. For the first year, the interest is $4,000 · 0.078 = $312, and for the second, it is
$2,000 · 0.078 = $156. The total interest is $312 + $156 = $468.

81 Percent (Blue Series)


82 Percent (Blue Series)
Math Mammoth has a variety of resources to fit your needs. All are available as economical
downloads, and most also as printed copies.

• Math Mammoth Light Blue Series


A complete curriculum for grades 1-7. Each grade level includes two student worktexts (A and
B), which contain all the instruction and exercises all in the same book, answer keys, tests,
cumulative reviews, and a worksheet maker. International (all metric), Canadian, and South
African versions are also available.
https://www.MathMammoth.com/complete-curriculum
https://www.MathMammoth.com/international/international
https://www.MathMammoth.com/canada/
https://www.MathMammoth.com/south_africa/

• Math Mammoth Skills Review Workbooks


These workbooks are intended to be used alongside the Light Blue series full curriculum, and
they provide additional review to the topics studied in the main curriculum, in a spiral manner.
https://www.MathMammoth.com/skills_review_workbooks/

• Math Mammoth Blue Series


Blue Series books are topical worktexts for grades 1-7, containing both instruction and
exercises. The topics cover all elementary mathematics from 1st through 7th grade. These
books are not tied to grade levels, and are thus great for filling in gaps.
https://www.MathMammoth.com/blue-series

• Make It Real Learning


These activity workbooks concentrate on answering the question, "Where is math used in real
life?" The series includes various workbooks for grades 3-12.
https://www.MathMammoth.com/worksheets/mirl/

• Review Workbooks
Workbooks for grades 1-7 that provide a comprehensive review of one grade level of math—for
example, for review during school break or summer vacation.
https://www.MathMammoth.com/review_workbooks/

Free gift!

• Receive over 350 free sample pages and worksheets from my books, plus other freebies:
https://www.MathMammoth.com/worksheets/free

Lastly...

• Inspire4 is an inspirational website for the whole family I've been privileged to help with:
https://www.inspire4.com

83 Percent (Blue Series)

You might also like